首页 2003-2011年全国大学生英语竞赛C类初赛、决赛真题及答案汇总

2003-2011年全国大学生英语竞赛C类初赛、决赛真题及答案汇总

举报
开通vip

2003-2011年全国大学生英语竞赛C类初赛、决赛真题及答案汇总2003-2011年全国大学生英语竞赛C类初赛、决赛真题及答案汇总 2003-2010年全国大学生英语竞赛C类初赛、决赛真题及答案汇总集(原创) 2003-2011年全国大学生英语竞赛试题 2011 National English Contest for College students (Level C-Preliminary) Part I Listening Comprehension (30 marks) Section A (5 marks) In this section, you wi...

2003-2011年全国大学生英语竞赛C类初赛、决赛真题及答案汇总
2003-2011年全国大学生英语竞赛C类初赛、决赛真题及答案汇总 2003-2010年全国大学生英语竞赛C类初赛、决赛真题及答案汇总集(原创) 2003-2011年全国大学生英语竞赛试题 2011 National English Contest for College students (Level C-Preliminary) Part I Listening Comprehension (30 marks) Section A (5 marks) In this section, you will hear five short conversations. Each conversations will be read only once. At the end of each conversation, there will be a twenty-second pause, read the question and the there choices marked A,B and C ,and decide which is the best answer.Then mark the corresponding letter on the answer sheet with a single line through the centre. 1. What does the man want to do? A. Get something to eat now. B. Find a quiet place that shows games. C. Watch the next game with the woman. 2. Why does not the man have a MySpace account? A. He is not skilled at using computer. B. All of the instruction are in English C. The woman won not teach him. 3. How long does the woman plan to try teleworking A. For a few days. B. For a few weeks. C. For a few months. 4. What does the man hope will happen? A. The price of cell phone novels will go down. B. The novel’s author will writer longer stories. C. The woman will tell him ho the story ends. 5. what is the woman going to do next? A. turn on her computer. B. Go for a walk with peter. C. Visit her new neighbors. 第 1 页 共 77 页 2003-2010年全国大学生英语竞赛C类初赛、决赛真题及答案汇总集(原创) Section B (10 marks) In this section, you will hear two long conversations. Each conversation will be read only once. At the end each conversation, there will be a one minute pause. During the pause, read the questions, each with three choices marked A, B and C, and decide which is the best answer. Then mark the corresponding letter on the answer sheet with a single line through the centre. Conversation one 6. What did Jack do over the summer? A. He studied very hard. B. He took a summer class. C. He visited one of his teachers. 7. What does Jack think of Ms Wellington as a teacher? A. Easy-going. B. Tough. C. Interesting. 8. Why is Ms Wellington’s class hard? A. Her exams are difficult. B. She does not give students the help they need. C. She makes do lots of work. Conversation two 9. Why is Mrs. Griffin going to the city where the hotel is located? A. He is on holiday. B. He’s on a business trip C. He is going to a conference. 10. How many times has Mrs. Griffin stayed at the Sunrise Hotel? A. Twice. B. Once. C. Three times. 11. Where is Mrs. Griffin form? A. Canada. B. New Zealand. 第 2 页 共 77 页 2003-2010年全国大学生英语竞赛C类初赛、决赛真题及答案汇总集(原创) C. Australia. 12. What is Mrs. Griffin’s passport number? A.87647489 B.87637289 C.87637489 13. What kind of room does Mrs. Griffin want? A. A single room for two nights. B. A double room for two nights. C. A single room for one night. 14. When will Mrs. Griffin arrive at Sunrise Hotel? A. at 9:15 pm B. at 9:35 pm C. at 10:00 pm 15. What food will be put into Mrs. Griffin’s room? A. a sandwich with fries. B. a cheese sandwich. C. a burger with chips. Section C (5Marks) 16. What does the Associated Press ask editor and news directors to do? A. vote for the top stories of the year. B. describe the oil spill in the Gulf of Mexico C. writes about the 11 workers killed in the explosion 17. Where are the doctors and technology experts from? A. New York. B. London C. Tokyo 18. For how long does President Obama agree to extend the tax cuts? A. for four years B. for three years C. for two years. 第 3 页 共 77 页 2003-2010年全国大学生英语竞赛C类初赛、决赛真题及答案汇总集(原创) 19. How many people in the world don’t have enough to eat,according to the report? A. more than one billion. B. some six hundred million. C. nearly nine hundred million 20. What have astronomers recently discovered? A. there are unknown plants in older galaxies. B. there are many galaxies in the universe C. there are a lot more red dwarf in older galaxies Section D (10 marks) In the section, you will hear a short passage. There are 10 missing words phrases. Fill in the blanks with the exact words or phrases you hear. Remember to write the answer on the answer sheet What do you do if you don’t get your first choice university? This ____ faces thousands of British every year. Many such_____ turn to Clearing, a service that helps find university places for students at the last moment. If they don’t have the marks to get into their____, Clearing tells them about places available at other university, though they might have to read a difficult subject. This year has seen a record number of people applying to university. This, combined with the _____________________,an uncertain job market, and budget cuts at university, product even more of a scramble for places than usual. Some sources say six students have applied for each remaining___________________________ place The British University Admissions Service, UCAS, says up to a quarter of this year’s university applicants-almost 190000 people-have not been admitted into a____________________________. That is an increase of over 46000 students from last year. Faced with these figure, some British students might consider an interesting alternative:_____________________________. The University of Nottingham for is offering place at its campuses in Ningbo, near Shanghai, and Kuala Lumpur, Malaysia. Students at these institutions can earn University of Nottingham degrees, according, engineering and English. Similarly, the University of Bolton says it has unlimited places at its campus in the United Arab Emirates. To deal with these problems, the UK’s Higher Education Minister, David Willet’ s, is 第 4 页 共 77 页 2003-2010年全国大学生英语竞赛C类初赛、决赛真题及答案汇总集(原创) encouraging students who have not made the grade to consider alternatives to university, such as _______________________and studying at home. “There are arrange of options available, “he says. “people can reapply next year, so they should consider spending this year in a way that will add positively to their CVs. Getting _____________________or other skills will strengthen their chances next year.” Some commentators say, though that rising university costs, poor long-term_______________________, and a drop in graduate recruitment mean this the worst time to be a university student in the UK. Part Two Vocabulary and Structure (15 marks ) There are 15 incomplete sentences in this section. For each blank there are four choices marked A, B, C and D. Choose the one that best completes the sentence. Then mark the corresponding letter on the answer sheet with a single line through the centre. 31. After four days of talks, we are glad to announce that the union and management have reached an______. The agreement is fair and benefits both sides. A. accord B. accomplishment C. identity D. undertaking 32. As the clerk______prepared my milk shake, I wondered how long she had been working there ,mindlessly making ice cream treats in a set order of steps. A. logically B. methodically C. graphically D. synthetically 33. As a boy he wanted to be a fireman. As a high school student, he thought he’d like to become a teacher. Now he______to be nothing more than a janitor. A. Assumes B. Prescribes C. Aspires D. Presumes 34. Regardless of what caused it, I an grateful that have finally reached a point in my life_______I can appreciate my strengths, accept my weaknesses and try to be comfortable with everything in between. A. Why B. Where C. Which D. What 35. ______information provided by members of the public, the police would have a much move difficult job. A. Supposing B. Provided the C. If it were not for D. On condition that 36. Peter Brown was a painstaking writer;______, he once spent half a day on the composition of a single sentence. 第 5 页 共 77 页 2003-2010年全国大学生英语竞赛C类初赛、决赛真题及答案汇总集(原创) A. On the other hand B. Nevertheless C. Moreover D. For example 37.----What an I going to do about a present for Carol? ----You______some flowers. A. Might have sent her B. Must have sent her C. Could send her D. Would send her 38. Without the air holding in some of the sun’s heat, the earth______cold at night, too cold for us to live on. A. Will be freezing B. Would be freezing C. An be frozen D. Would be frozen 39. The students in our university each______an English dictionary. That is to say, each of the students in our university______an English dictionary. A. Have; has B. Have; have C. Has; have D. Has; has 40. Here’s your kitchen. I hope you enjoy cooking here. Is there______else that you need? A. Something B. Anything C. Nothing D. Everything 41. David______his business partner over plans to reduce the workforce. A. Came down to B. Broke down to C. Fell out with D. Went along with 42. ______is this piece of equipment to be removed from the building. A. On no account B. Absolutely C. Scarcely D. Not at all 43. Helen’ s parents were______that she was still on the job., but she had resigned. A. In doubt B. Of the opinion C. Under the impression D. With suspicion 44.----I don’t think I will ever, in my life, win a lottery of five million dollars. ----Well, ______. Anything can happen. A. You made it B. You’re kidding C. What you say D. You can never tell 45.-----How did you find the concert in the Grand Theatre last night? -----______ but the conductor was perfect. A. I couldn’t agree more B. I didn’t think much of it C. I was crazy about it D. I really liked it Part Three Cloze(15 marks ) 第 6 页 共 77 页 2003-2010年全国大学生英语竞赛C类初赛、决赛真题及答案汇总集(原创) I have been reading a lot on my iPad recently, and I have some (46)_____ (complain) not about the iPad itself but about the state of digital reading generally. Reading is a subtle thing, and its subtleties are artifacts of a venerable medium: words printed in ink on paper. Glass and pixels aren’t the same. When I read a physical book, I don’t have to look anywhere else to find out how much I’ve read. The iPad e---reader, iBooks tries to create the (47) illu_____ of a physical book. The pages seem to turn, and I can the edges of those that remain, but it’s fake. There are always exactly six unturned pages, no matter (48)_____ I am in the book. Also, there is a larger problem. Books in their digital format look vastly less "finished", or less genuine than real books. You can vary their font and type size, but this only makes them (49)_____(resemble) word---processed---no matter how (50)_____(wretch) or wonderful they are---will never look as good as Robert Hass’s poems in the print edition of The Apple Trees at Olema. But your poems can look almost exactly as ugly---as "e---book---like" ---as the Kindle version of that collection. All the e---book I’ve read have been ugly---books by Chang---rae Lee, Alvin Kernan, and Stieg Larsson---though the texts have been wonderful. I didn’t grow up reading texts. I grew up reading books, and this(51)_____(differ) is important. When it comes to digital editions, the(52)_____(assume) seems to be that all books(53)are_____(create) equal. However, nothing could be further from the truth. In the mass migration from print to digital, we’re seeing a profusion of digital books---many of them out of copyright---that look new and even "HD," but which may well have been supplanted by more accurate editions and better translations. We need a digital readers’ guide---a place where readers can find(54)_____ whether the book they’re about to download is the best available edition. (55)Fi_____, two related problems. I already have a personal library, but most of the books I’ve read have come from(56)_____(lend) libraries. Barnes & Noble has released an e---reader that allows short---term (57) _____(borrow) of some books. The entire idea behind Amazon’s Kindle and Apple’s iBooks assumes that you cannot read a book unless you own it first and that only you can read it unless you want to give your reading device to someone else. This goes against the social value of reading, the collective knowledge and (58)_____(collaborate) discourse that comes from access to (59)_____or our culture in general. 第 7 页 共 77 页 2003-2010年全国大学生英语竞赛C类初赛、决赛真题及答案汇总集(原创) Part Four Reading Comprehension (40 marks). Section A (10 marks ). Questions 61 to 65 are based on the following passage. Not keen on reading? Do you have trouble finding a novel that arouses your interest? Why not follow Ammon Shea’s example and start reading a dictionary? Mr Shea owns over 1,000 dictionaries and he reads them for fun. He recently spent a year reading all 20 volumes of the Oxford English Dictionary. The dictionary contains more than 20,000 pages and over 59 million words. As he read from A to Z, he noted down interesting words in a ledger. This includes words such as "happify," meaning to make someone happy and "tripudiate", which means to dance, skip or leap for joy. Mr Shea also kept a diary about this experience, which has since become a best---selling book. Why did he do this? He claims it was fun. "I’ve always enjoyed reading dictionaries . They are far more interesting than people give then credit for," he said. It appears that it was not his goal to sound more intelligent by using longer and more complex words. "I’m not against long, fancy or obscure words, but I’m opposed to using then for their own sake," he said. In fact ,as a result of reading so many new words , Mr Shea often forgot everyday vocabulary. He wrote, "My head was so full of words that I often had trouble forming simple sentences." Mr Shea is not alone in his love of reading dictionaries.Elaine Higgleton, a representative of Collins Cbuild dictionaries, explained that thousands of crossword puzzle and Srabble fans read dictionaries for fun and to improve their games. Ms Higgleton did however note that, "It’s probably not the best way to learn English ,and you’d learn more than you need." It is not known how many of the 59 million words Ms Shea remembers, but he has certainly made history with his eccentric hobby. Questions 61 to 65. 第 8 页 共 77 页 2003-2010年全国大学生英语竞赛C类初赛、决赛真题及答案汇总集(原创) Decide whether the following statements are True or False. 61. Mr Shea has read 1.000 dictionaries. 62. Mr Shea spent one month reading the Oxford English Dictionary. 63. In Mr Shea’s opinion,people don’t give dictionaries enough credit for being interesting. 64. Mr Shea thinks it is important t be able to use long and complicated words in everyday conversation. 65. Elaine Higgleton thinks that reading a dictionary is the best way to learn English. Section B (10 marks) Questions 66 to 70are based on the following passage. Surfing is something people often get hooked on after trying it a few times. For many surfers it is much more than a hobby---they would probably agree with the American professional surfer Kelly Slater when he said,"Once you’re in, you’re in. There’s no getting out." "Surfing", of course, refers to riding on ocean waves using a surfboard. Many surfers stand up on their boards, which requires god balance and is therefore difficult for most beginners to learn, but some lie down and "bodyboard" The history of surfing probably began with the Polynesian people of the Pacific Islands. One of the first white people to see anyone surfing was the British explorer Captain Gook, when his ship arrived in Hawaii in 1779. He watched many Hawaiians riding waves on large pieces of wood, and reported that, "Surfing seems to give them a feeling of great pleasure. " When surfing started to become very popular in the United States in the 1950’s and 60s, surfers used large wooden boards (often more than three metres long) that were quite heavy. Boards today are shorter and also much lighter, because they are made of artificial materials instead of wood. For anyone who wants to try surfing. The only essentials are waves and a board. There are a few other things, however, that most surfers find important; a cord t attach one of their ankles to the board and therefore stop it from being carried a long way away when they fall off’; wax, which they put on the surface of the board to help their feet stick to it; and a wetsuit to help them keep warm in cold water. The south---west of English is an example of a place where surfers usually need wetsuits, even in summer. Surfing has been a professional sport for many yeara and the very best surfers are able to make 第 9 页 共 77 页 2003-2010年全国大学生英语竞赛C类初赛、决赛真题及答案汇总集(原创) a living from it. Most of the best professional surfers in the last 30 years, both men and women, have been American or Australian, but surfers from Brazil, Peru and South Africa have also won important competitions. Questions 66 to 70 Answer the following questions with the information given in the assage in a maximum of 10 words for each question. 66. Why do most beginners find it difficult t stand up on a surfboard? 67. In what part of the world did surfing probably begin? 68. When did surfing start to become very popular in the United States? 69. What do surfers use wax for? 70. According to the passage, in what part of the world do surfers usually need wetsuits? Section C (10 marks) Questions 71 to 75 are based on the following passage. The latest human development report from the United Nations Development Programmed (UNDP) contains some good news, but also a very serious warning about the threat posed y climate change. The report, published annually since 1990, seeks to asses “human development” around the world, and calculates a “Human Development Index (HDI) for 169 counties. The HDI is based on average income, life expectancy and level of education in a country. Not surprisingly, rich counties tend to have higher HDIs than poor counties, but there are interesting variations in human development among countries with similar levels of economic development, because some have better health and education systems than others. According to the 2010 report, the county with the highest level of human development is Norway, followed by Australia, New Zealand, the United States and Ireland. Most of the lowest HDIs belong to counties in sub-Saharan Africa. Almost all counties around the worlds have higher HDIs now than in 1990, despite the fact that since the 2008 financial crisis, the total number of people living in extreme poverty has increased. The report concludes that most people are healthier, live longer, are better educated and have access to more goods and services. Even in countries with severe economic problems, people’s level of health and education as generally improved. Although sub-Saharan African 第 10 页 共 77 页 2003-2010年全国大学生英语竞赛C类初赛、决赛真题及答案汇总集(原创) countries are at the bottom of the pile in terms of human development, some of them have made significant progress since 1990. The report is critical, however, of the fact economic inequality has increased significantly in the last twenty years, both within and between countries. The greatest threat to improving HDIs in the future, according to the report, is climate change. Economic growth increases average incomes in a country through increasing production and consumption. However , if this leads to greater emissions of greenhouse gases, as has always been the case in the past, global warning will probably accelerate, and cause severe environmental problems in some parts of the world hat will threaten the livelihoods of huge numbers f people. The progress of the last twenty years, therefore, might not be sustainable. The only solution, according to the report, I to break the link between economic growth and greenhouse gas emissions-which, needless to say, is easier, said than done. Questions 71 to 75 Complete the following sentences with information given in the passage in a maximum of 10 words for each blank. 71. The concept of “human development” is based on the following three factors: _______, _____________and_____________. 72. Some countries with similar levels of economic development have quite different HDIs because they have_____________. 73._________________has caused the number of people living in extreme poverty to increase since 2008. 74. The report says that ______________ is the greatest threat to increasing HDIs in the future. 75. The report says the link between____________ and ____________needs to be broken. Section D (10 marks) Questions 76 to 80 are based on the following passage It is natural for young people to be critical of their parents at times and to blame them for most of the misunderstanding between them. They have always complained, more or less justly, that their parents are out of touch with modern ways; that they are possessive and dominant; that they do not trust their children to deal with crises: that they talk too much about certain problems-and that they have no sense o humor, at least parent-child relationships. I think it is true that parents often underestimate their teenage children and also forget how 第 11 页 共 77 页 2003-2010年全国大学生英语竞赛C类初赛、决赛真题及答案汇总集(原创) they felt themselves when were young. Young people often irritate their parents with their choices in clothes, hairstyles, entertainers and music. This is not their motive. They feel cut off from the adult world into which they have not yet been accepted, so they create a culture and society and their own. Then, if it turns out that their music, entertainers, vocabulary, clothes or hairstyles irritate their parents, this gives them additional enjoyment. They feel they are superior, at least in a small way, and that they are leaders in style and taste. Sometimes teenagers are resistant and proud because they do not want their parents to approve of what they do. If they did approve, it looks as if the teenager is betraying his own age group. All this is assuming that the teenager is the underdog: he can not win but at least he can keep his honor. This is a passive way of looking at things. It is natural enough after years of childhood, when children were completely under their parent’s control, but it ignores the fact that when they become teenagers, children are beginning to be responsible for themselves. If you plan to control your life, co-operation should be a part of that plan. You can charm other people, especially your parents, into doing things the way you want. You can also impress people with your of responsibility and your initiative, so that they will give you the authority to do what you want to do. Questions 76 to 78 Choose the best answer according to the passage. 76 the first paragraph is mainly about_____________. A teenagers’ criticism of their parents B misunderstanding between teenagers and their parents C the dominance of parents over their children D teenagers’ ability to deal with crises 77 teenagers have strange clothes and hairstyles because they___________ A have a strong desire to be leaders in style and taste B want to prove their existence by creating a culture of their own C have no other way to enjoy themselves D want to irritate their parent 78 teenagers do not want their parents to approve of what they do because they 第 12 页 共 77 页 2003-2010年全国大学生英语竞赛C类初赛、决赛真题及答案汇总集(原创) _______________. A have already been accepted into adult world B feel that they are superior to adult world C want to win adults over to their culture D don’t want to appear to be disloyal to their own age group Question 79 to 80 Translate the sentences in the passage into Chinese 79 I think it is true that parents often underestimate their teenage children and also forget how they felt themselves when were young 80 You can charm other people, especially your parents, into doing things the way you want. Part V translate (10 mark) Translate the following sentences into English, using the words given brackets. Remember to write your answer on the answer sheet. 81 他以牺牲健康为代价获得财富。(at the cost of ) 82 我不知道有多少学生涉及到这次事件。(be involved in ) 83 为孤儿捐款,请按以下地址或电话同我们联系。(contact) 84 目前,在韩国和日本, 汉语已成为第二外语,仅次于英语。(second to) 85 要不是下雨,我们的旅行本来是很愉快的。(but for) Part VII writing (30 marks) Task I (10marks) It’s the beginning of a new year. Write describing resolutions you make to try to change your life and why you Li Ming instead Write a note of about 100 words in an appropriate style on the answer sheet. Task II (20marks) Write about the following topic. Success is something most people try to achieve. What does success mean to different people, and what is your view or success? Give reasons for your answer and include any relevant example from your own knowledge or experience. Write your article in about 140 words in an appropriate style on the answer sheet. 第 13 页 共 77 页 2003-2010年全国大学生英语竞赛C类初赛、决赛真题及答案汇总集(原创) 第 14 页 共 77 页 2003-2010年全国大学生英语竞赛C类初赛、决赛真题及答案汇总集(原创) 第 15 页 共 77 页 2003-2010年全国大学生英语竞赛C类初赛、决赛真题及答案汇总集(原创) 第 16 页 共 77 页 2003-2010年全国大学生英语竞赛C类初赛、决赛真题及答案汇总集(原创) 2003年全国大学生英语竞赛初赛试题 2003 National English Contest for College Students (Preliminary) Part I Listening Comprehension(30 minutes,30 points) Section A Dialogues(10 points) Directions:In this section ,you will hear 10 short dialogues(At the end of each dialogue,a question will be asked about what was said(Both the dialogue and the question will be read only once(After each question there will be a pause(during the pause,you must read the four choices marked A,B,C and D,and decide which is the best answer(Then m ark the corresponding letter on the Answer Sheet with a single line through the center( 1(A(A sales clerk( B(A police officer( C(A tailor( D(A nurse( 2(A(By train( B(She walks( C(By car( D(By bus( 3(A(Fish is the only dish left( B(Chicken is the only dish left( C(Vegetarian meals are not offered( D(There aren’t any vegetarian meals left( 4(A(He starts work next weekend( B(He’ll be away( C(He’ll be in the mountains( D(He’s moving to Florida( 5(A(In an elevator( B(At a dress store( C(On the seventh floor( D(At a department store( 6(A(They felt it was disorganized( B(They were pleased with its Asian content( C(They felt it lacked Asian content( D(They felt it ignored recent events( 7(A(He doesn’t have enough time( B(He doesn’t have a watch( C(The library doesn’t have the articles he wants( D(He can’t find the library( 8(A(He wants the woman to dine out with them( B(He wants to work tomorrow( C(He wants the woman to finish dinner first( D(He wants to pay for the dinner( 9(A(Twice a day( B(Twice a week( C(Once a week( D(Daily( 10(A(At two o’clock( B(At four o’clock( C(At three thirty( D(At eight o’clock( Section B News Items(10 points) Directions:In this section,you will hear 10 pieces of short news from BBC or VOA(There will be a question following each piece of news(Write down the answer to each question in no more than 15 words( 11(_______________________________________ 12(_______________________________________ 第 17 页 共 77 页 2003-2010年全国大学生英语竞赛C类初赛、决赛真题及答案汇总集(原创) 13(_______________________________________ 14(_______________________________________ 15(_______________________________________ 16(_______________________________________ 17(_______________________________________ 18(_______________________________________ 19(_______________________________________ 20(_______________________________________ Section C Compound Dictation(10 points) Directions:In this section,you will hear a passage three times(When the passage is read for the first time,you should listen carefully for its general idea(Then listen to the passage again(When the passage is read for the second time,you are required to fill in the blanks numbered from 21 to 28 with the exact words you have just heard(For blanks numbered from 29 to 30,you are required to fill in the missing information(You can either use the exact words you have just heard or write down the main points in your own words(Finally,when the passage is read for the third time, you should check what you have written and rewrite the correct answers on the Answer Sheet( Although general Motors and General Electric are large multinational companies with operations around the globe,there are numerous smaller companies that engage in international trade(Because 95percent of the world’s population and two-thirds of its (21)_____ power are located outside the United States,it is important for American (22)_____to be present in foreign markets(However,before we explain the different methods by which a company may (23)_____in international trade,we might first consider some important (24)_____that U.S(companies often fail to study before they sell products in a foreign country(These factors are (25)_____with differences in language,in values and attitudes,and in political (26)_____( When (27)_____Coca-Cola into the Chinese market in 1920,the company used a group of Chinese symbols that,when spoken,sounded like Coca-Cola(However, when read, these symbols meant,“a female horse fattened with wax”(Upon reentering the Chinese market in the 1970s,Coca-Cola used a series of Chinese (28)_____that translates into“happiness in the mouth”((29)_________________________( Culture is the total pattern of human behavior that is practiced by a particular group of people((30)_________________________( Part II Vocabulary and Structure(15 minutes,30 points) Section A Multiple Choice(20 points) Directions:Questions 31-50 constitute a complete passage(There are 20blanks in the passage(For each blank there are four choices marked A,B,C and D(Choose the one that best completes the sentence(Then mark the corresponding letter on the Answer Sheet with a single line through the centre( 31(Senior Metropolitan police officers tried to dismiss the Noting Hill race riots which raged for five nights over the August bank holiday in 1958 as the work of“ruffians ,both colored and white”hell-bent on hooliganism ,according to _____ official files( A(recent revealed B(newly released C(previous disclosing D(earlier exposing 32(But police eyewitness reports in the secret papers_____ that they were overwhelmingly the work of a white working class mob out to get the“niggers”( 第 18 页 共 77 页 2003-2010年全国大学生英语竞赛C类初赛、决赛真题及答案汇总集(原创) A(contain B(convince C(consist D(confirm 33(The ferocity of the Noting Hill“racial riots”,as the press called them at the time,shocked Britain into_____ for the first time that it was not above the kind of racial conflict then being played out in the American deep south( A(realizing B(witnessing C(watching D(identifying 34(The carnival,which will_____ the streets of west London _____more than 1.5 million people this weekend,was started in 1959 as a direct response to the riots( A(crowd;of B(pour;for C(fill;with D(emerge;in 35(While senior officers tried to play down the racial aspects of the riots,the internal Metropolitan police files released this month at the public record office confirm that the disturbances were overwhelmingly _____ by 300 to 400 strong“Keep Britain White” mobs ,many of them Teddy boys armed with iron bars ,butcher’s knives and weighted leather belts,who went“nigger-hunting”among the West Indian residents of Noting Hill and Noting Dale( A(erupted B(commenced C(triggered D(inaugurated 36(The first night left five black men _____ on the pavements of Noting Hill( A(lying unconscious B(there died C(feel faint D(serious hurt 37(The battles raged over the bank holiday weekend as the black _____responded in kind with counterattacks by large groups of“men of color”similarly armed( A(column B(army C(brigade D(community 38(Thomas Williams was stopped by the police as he came out of Bluey’s Club on Talbot Road,Noting Hill(He _____a piece of iron down his left trouser leg,a petrol bomb in his right pocket and a razor blade in his inside breast pocket:“I have to protect myself,”he told the arresting officer( A(found to have B(was found to have C(found having D(was found having 39(The _____ files,which were sealed under the 75-year rule but have been released early,show that senior officers tried to convince the then home secretary, “Rab”Butler,that there was not a racial element to the rioting( A(forbidden B(confidential C(incredible D(strict 40(In his official report,Detective Sergeant M.Walters of the Notting Hill police said the national press had been wrong to portray the“widespread series of street disturbances”as“racial”riots:“Whereas there certainly was some _____ feeling between white and colored residents in this area,it is abundantly clear much of the trouble was caused by ruffians,both colored and white,who seized on this opportunity to indulge in hooliganism (” A(ill B(sick C(painful D(hurt 41(But the police witness statements and private statistics _____ ( 第 19 页 共 77 页 2003-2010年全国大学生英语竞赛C类初赛、决赛真题及答案汇总集(原创) A(told differently B(interpreted in a different way C(existed m any differences D(told a different story 42(The Met com missioner was told that _____ the 108people who were charged with offences ranging from grievous bodily harm to affray and riot and possessing offensive weapons,72 were white and 36 were “colored”( A(for B(from C(of D(in 43(It is popularly believed that the riot began on the night of Saturday,August 20, when a 400-strong crowd of white men,_____“Teds”,attacked houses occupied by West Indians( A(they are all B(many of them C(some were D(most of them belong to 44(Among the _____ was Majbritt Morrison ,a young white Swedish bride of a Jamaican( A(offenders B(rioters C(victims D(residents 45(She was pelted with stones,glass and wood,and _____ in the back with an iron bar as she tried to get home( A(bruised B(struck C(patted D(scratched 46(The internal police witness statements provide graphic evidence of the motives of the mobs—at one point crowds several thousand strong roamed the streets of Notting Hill,_____ homes and attacking any West Indian they could find( A(plunging into B(breaking into C(seeking for D(searching for 47(PC Richard Bedford said he had seen a mob of 300 to 400 white people in Bramley Road _____:“We will kill all black bastards(Why don’t you send them home,” A(shouting B(to cry C(utter D(announced 48(PC Ian McQueen on the same night said he was told:“Mind your own _____, cops(Keep out of it(We will settle these niggers our way(We’ll murder the bastards(” A(matters B(affair C(things D(business 49(The disturbances continued night after night until they finally petered out on September 5(At the Old Bailey Judge Salmon later handed down exemplary _____ of four years each on nine white youths who had gone“nigger hunting”( A(decisions B(statements C(trials D(sentences 50(While those dealt with by the courts were overwhelmingly white ,the large number of black people also arrested and the official _____ there had not been a racial motive ensured a legacy of black mistrust of the Metropolitan police that has never really been eradicated( A(persistence B(perseverance C(insistence D(instance Section B Error Correction(10points) Directions:The following passage contains 9 errors(In each case only one word is involved(You should proofread the passage on the Answer Sheet and correct it in the following way: 第 20 页 共 77 页 2003-2010年全国大学生英语竞赛C类初赛、决赛真题及答案汇总集(原创) EXAMPLE One night,quite late,I was still awake in the room I am shared with 1. am my husband(I was lying on my right side and can hear a child crying( 2. could Getting up,I went ? see if our son was all right( 3. to He was sleeping soundly,breathing deeply and gently( 4. ? The Zipper Whatever did we do before the invention of the zipper, In 1893 the world’s first zipper was produced in Chicago( Although the inventor claimed that it was a reliable fastening for clothing,this was not the case(The Chicago zipper sprang 51(______ open without warning,or jammed shut,and it swiftly lost popularity(Twenty years ago a Swedish-born engineer called 52(______ Sundback solved the problem(He attached tiny cups to the backs of the interlocked teeth,and this meant that the teeth 53(______ could be enmeshed more firmly and reliably( At first zippers were made of metal(They were heavy,and if they got stuck it was difficult to free(Then came nylon 54(______ zippers which were lighter and easier to use,and had smaller teeth(The fashion industry liked the new zippers far better because they didn’t distort the line of the garment or weighing 55(______ down light fabrics(They were also easier for the machinists to fit into the garment( Meanwhile a new fastening agent made its appearance at the end of the twenty century: velcro. Velcro is another product 56(______ made from nylon(Nylon is a very tough synthetic fibre first developed in the 1930s,and bearing a name to mind the wearer 57(______ of the two places where it was developed:NY for New York and LON for London(Velcro is made with very small nylon hooks on one side of the fastening which caught tiny looped whiskers on the 58(______ other side of the fastening(It is strong and durable( Velcro is used on clothing,luggages and footwear(It is quick 59(______ and easy to fasten and unfasten,and has taken a large part of the zipper’s share of the market(It is also used in ways a zipper cannot be used—for instance as an easily changed fastening on plaster casts,and to hold furnishing fabrics in a position( 60(______ Part III Situational Dialogues(5 minutes,10 points) Directions:Complete the following dialogues by choosing the best answer(Then mark the corresponding letter on the Answer Sheet with a single line through the centre( 61(Rob:Hey Jill,you’re looking great( Jill:Thanks,Rob(____________ Rob:Well,you did it(How, Jill: I jog every morning,and I go to aerobics every other day( A(I bought this dress yesterday(Really smart( B(You are looking fine too( 第 21 页 共 77 页 2003-2010年全国大学生英语竞赛C类初赛、决赛真题及答案汇总集(原创) C(I’m recovering my strength after the flu( D(My New Year’s resolution was to get in shape( 62(Bob:Hi Jane(How are you, Jane:____________I didn’t sleep a wink last night(The people next door were making a lot of noise again till very late at night( A(I’m feeling a bit out of sorts this morning( B(Fine,thank you(And you, C(I slept like a log and didn’t want to get out of bed( D(It seems a bit unusual,you know( 63(Ann :Aah~He’s gorgeous~Look at those big,golden paws(When did you get him, Roger:Yesterday(____________ Ann :Oh,right(What kind is she, Roger:A Labrador( A(Susan’s got a more beautiful one( B(What’s up, C(It’s a she actually( D(Isn’t it right, 64(Tina:Wow,look at all the things on sale(____________ Andrew:Yes,look,this shirt is 50 ,off( Tina:And look at these shoes(They are 30 ,off the normal price( A(I’d like to buy a skirt(B(There are some real bargains( C(Are the prices reasonable,D(These shoes are the same as mine( 65(Woman:Have you finished the packaging, Man :____________ Woman:Good(Because the truck will be coming soon,this is a rush job( A(Don’t hurry m or I’ll break the glass( B(Almost(I just have to wrap the glass and put it into boxes( C(No,I haven’t(Why didn’t you help me with it, D(Yes,I have(What else can I do for you, 66(Customs Officer :________________________ Mrs(John son :No,nothing at all( Customs Officer :No perfume,alcohol or cigarettes, Mrs(John son :Well,I have 200 cigarettes;that’s all( A(Do you have anything in the bag,ma’am, B(Do you have anything to declare,ma’am, C(Do you want to buy something,ma’am, D(Is there anything I can do for you,ma’am, 67(Linda:Hello(I’d like to send this package,please( Clerk:____________________________________ Linda:First class(How long will that take, Clerk:About three days( A(How would you like to send it, B(Which class are you in, C(Where do you want to send it to,D(Which class is it in, 68(Assistant:Can I help you, 第 22 页 共 77 页 2003-2010年全国大学生英语竞赛C类初赛、决赛真题及答案汇总集(原创) Colin :Yes,it’s about this sports shirt(I washed it the other day(The colour ran and it shrank( Assistant:Oh dear,I see(________________________ Colin :I’m afraid not( Assistant:I’m sorry,but I’m not allowed to change anything without a receipt( A(Did you buy it here, B(Would you want to change it, C(Do you have the receipt, D(Could you tell me who sold it to you, 69(James:Could I have my bill,please,Can I pay by credit card or eurocheque, Receptionist:____________ James:I’ll pay by credit card,then( Receptionist:That’s fine(I hope you enjoyed your stay here( A(Here’s your bill(B(Sorry,we don’t take credit card( C(You can pay by eurocheque(D(Yes,we take both( 70(Husband:When is our anniversary, Wife:________________________ Husband:No,it’s just that I bought these flowers for you and I was hoping today was the day( A(Hmm (((I can’t remember either(Why, B(Hey,are these flowers for me, C(Who cares,Do you want to give me a surprise, D(Are you joking,Have you really forgotten again, Part IV Reading Comprehension(25 minutes,40 points) Section A Multiple Choice(10 points) Directions:There is one reading passage in this part(The passage is followed by 5 questions or unfinished statements(For each of them there are four choices marked A,B,C and D(You should decide on the best choice and mark the corresponding letter on the Answer Sheet with a single line through the centre( Questions 71 to 75 are based on the following passage: Taking a peep at what’s going on in your head CARL Filer,18,a star salesman at a B,Q hardware store in the UK,was called up for promotion within one week of starting work(But,instead of being made supervisor,he was sacked—after his employers saw the results of his psychometric test( over-achiever is asking for trouble,but Mr Filer already thought he had proved himself more than capable( This year,nearly half of UK firms—46 percent—will use psychometric tests to select trainees,compared with just 17 percent in 2000, according to a report for GTI,a publisher of graduate career guides( These tests,which rate candidates’ ability and gauge their personality,have been used in the UK since the 1980s(But assorted studies have shown most people—graduates in particular—are wholly cynical about the idea of their personality being“measured( “People tend to see them as either too silly or too clever,, professor of occupational psychology at University of London(“But all the evidence indicates the tests do have 第 23 页 共 77 页 2003-2010年全国大学生英语竞赛C类初赛、决赛真题及答案汇总集(原创) some value( The first personality test as we know it,was developed by the American army in 1917 to filter out weak recruits( But it was not until the 1980s that the tests became popular in Britain(With a rising number of graduates going for a decreasing number of jobs,organizations began to see psychometric testing as a cheap,reliable alternative to the expensive, time-consuming interview( But today the tests are becoming alarmingly sophisticated and are edging towards probing the“dark side:pathology and personality disorders(Increasingly,tests are being used to try to detect promising young graduates who may,later in life,fly off the rails(go crazy);or to stop psychopaths(having mental disorder)getting recruited( In the future,interviewees could even be given a mouth swab to reveal the genetic and biological markers of personality(“We are heading for the era of genetic screening,”warns Carolyn Jones,of the Institute for Employment Rights(“I think these tests are very flawed( And there are other problems with the tests(For starters,it is possible to fake it—even the test producers agree on this(But they have made it as hard as possible(For example, look at whether you agree or disagree with the following two statements: difficult(, The main argument,however,is that the tests are invalid and cannot quantify(put a numerical value on)something as changeable as personality( The golden rule is then,that a psychometric test should never be used as the sole basis of selection,but should always be followed by interviews( 71(Most people’s attitude towards the psychometric test is ______( A(contemptuous B(favorable C(tolerant D(confounded 72(Which of the following is one of the reasons why psychometric testing wins an advantage over interviews, A(It doesn’t cost any money( B(It requires no equipment( C(It is time-saving( D(It can be done within seconds( 73(Which of the following statements is the author’s idea, A(Psychometric tests are defective( B(Psychometric tests should not be the only way to recruit promising young graduates( C(Psychometric tests are invalid and cannot quantify something changeable as personality( D(Psychometric tests are golden rules( 74(The test producers make the tests very complicated to ______( A(avoid cheating B(improve genetic screening C(find out the best ideas D(generate new concepts 75(Which of the following is not true according to the passage, A(The American army developed the first personality test to screen out weak recruits( B(In the future,interviewers could give a mouth swab to reveal interviewees’ symptoms( C(There are possibilities for starters to cheat in the psychometric tests( D(Interviews still play an important role in evaluating interviewees( Section B Short Answer Questions(30 points) Directions:In this part there are 3 passages with 15 questions or incomplete statements(Read the passages carefully(Then answer the questions in the fewest possible words(not exceeding 10 第 24 页 共 77 页 2003-2010年全国大学生英语竞赛C类初赛、决赛真题及答案汇总集(原创) words)(Remember to rewrite the answers on the Answer Sheet( Questions 76 to 80 are based on the following passage: The 8 Steps of Social Invention 1(Get ready to play( Like other types of creativity,social inventiveness flourishes when you begin thinking outside conventional boundaries(Charlie Girsch,a St(Paul,Minnesota-based creativity consultant,suggests that you start by playing with obviously absurd explanations for everyday events(“If traffic is slow,you’ll be tem pted to say, ‘Hmm(Must be an accident up ahead(’ Instead,try saying,‘Must be a family of turtles crossing the highway’or‘I expect there’s some kind of alien abduction going on(’You’ll be amazed how soon you will be looking at familiar problems in new ways(”Girsch’s book,Fanning the Creative Spirit(Creativity Central,1999)has scores of other exercises for limbering up the inventive part of your brain( 2(Generate a zillion far-fetched ideas( Concerned about the homeless in your neighborhood,Imagine a Homeless Parliament,a Homeless Circus,homeless families forming an orchestra,a homeless museum (((and on and on(Generate like mad with no regard for feasibility in order, as social invention pioneer Nicholas Albery advises,to“overcome e worthy-but-dull ideas(”Eventually the two or three best ideas will begin to stand out( 3(Take your wildest idea and bring it down to earth( How about that Homeless Circus,Could it turn into a forum for homeless people to display their creative talents,A performance series about homelessness,A neighborhood carnival with the homeless as guests of honor,Your flakiest idea may have a germ of brilliance that actually makes it more attractive,and thus more feasible (and fundable),than its worthy-but-dull cousins( 4(Look for in venations that solve more than one problem( The Slow Food Movement,born in Italy,boosts local farmers and regional cuisine traditions and restaurateurs and the same time that it“feeds”our hunger for authentic tastes, healthy eating,and a more leisurely,saner style of life( 5(Accentuate the positive( “A very common question that I get when I work with people in communities is ‘Why doesn’t anybody care about our problems,’”notes M chael Patterson,a social inventor and activist in Massachusetts(“What a worthless question(‘Why’, questions are for philosophers(Ask‘How’,and‘What’,questions—they are a lot more practical(”For instance, Patterson asks,“What would you do if you knew you couldn’t fail,” 6(Give it a rest( Walk away from your favorite idea for a while,forget about it,let it sleep(With your conscious mind out of the way,your subconscious gets to fiddle with the concept for a while, and you just might have an unexpected insight or breakthrough( 7(Practice“yes and”in stead of“yes but”( No matter how tempted you are to say“Yes,but this will be hard because,”or“Yes, but a million other people are doing this,”shift the conjunction to“and”and see what sort of positive refinement or change emerges(“Yes,and we could concentrate on immigrants(”“Yes,and we can make it open to all ages(” 8(Get your idea into the world( This is the tough part(You might seek out the help of activists who will take a shine to your 第 25 页 共 77 页 2003-2010年全国大学生英语竞赛C类初赛、决赛真题及答案汇总集(原创) ideas(Or become an organizer yourself(Paul Glove,a New York social inventor, coun-sels:“If you have an idea you believe in,write a pamphlet with your phone number on it and post it in Laundromats and bookstores(If three people call you,have lunch with them and call yourselves an organization(If five people call,meet with them and issue a press release(” Presto,you’re launched( 76(To generate far-fetched ideas helps to ______( 77(Michael Patterson wants us to come up with“How”,and“What”,instead of“Why”,questions because he considers they are more practical than ______( 78(The purpose to practice“yes and”instead of“yes but”is to make yourself more ______( 79(According to the article,when one has difficulty developing his favorite idea, he should ______( 80(One should not only generate far-fetched ideas but also ______ because the latter step is the nearest to reality( Questions 81 to 85 are based on the following passage: Thin Slice of TV Has Big Market It is too early to write an obituary for bulky picture tubes,which will remain the most affordable TV sets for years to come( But,analysts and industry executives insist that thin screens already have started to become the dominant format for TV sets in the digital era( Sharp price cuts have brought plasma sets and other thin,flat televisions out of high-end electronic boutiques and into thousands of mass-market outlets such as Cosco,a wholesale buying club in the US,best known for offering members bulk items and big discounts( The least expensive plasma sets still cost a hefty US ,3,000or more ,yet sales are growing so rapidly that many manufacturers are racing to boost production( That increase,combined with expanding production capacity and improved technology, could push the price of plasma sets down by one-third next year, according to analyst Richard Doherty of Envisioneering Group,a US research firm( But manufacturers are not just competing with each other;they are also trying to fend off challenges from competing thin-screen technologies,such as liquid crystal displays(LCD)( The demand for thin screens is fuelled in part by the advent of DVDs and digital TV broadcasts,which offer more detailed pictures and more lifelike colors than conventional analog TV signals( To see the difference,consumers need a set that can pack more information onto the screen than their current TVs can( This sharpness is most vivid on screens that are 40inches diagonal or larger(At that size,however,traditional direct view and projection TVs are so bulky that many consumers have trouble finding a place for them at home( Hence the interest in thin screens—models slender and light enough to hang on a wall( The glass panels at the heart of plasma and LCD sets come mainly from about a dozen companies with factories in Japan,South Korea and,increasingly, China(About 800,000 plasma panels will be shipped this year around the world, analysts say( That is a tiny amount compared with the overall market for TVs,which was about 140 million sets last year(But,industry experts said 2003would be a“breakout year” or plasma because shipments should double( Helping drive the growth are new or expanded manufacturing facilities(For example, Japanese 第 26 页 共 77 页 2003-2010年全国大学生英语竞赛C类初赛、决赛真题及答案汇总集(原创) electronics giant NEC last year doubled the capacity of its Japanese factory—reaching 300,000to 400,000 plasma panels(And it plans to double it again in 2003,officials said( As competition has heated up during the last four years,prices have fallen more than 50 percent(According to“NPD Tec world”,the average price of a plasma display sold in the US dropped from US ,12,700in January 1999 to US ,6,100in October 2002( The best markets for plasma screens have been in Asia,and about half of the sets have gone to businesses instead of homes( LCD TVs carry a premium price—they can be 10 times as expensive as a comparable tube-driven television—that knocks them out of most buyers’ budgets( But LCD panels are quickly taking over the market for computer monitors,and the tens of millions of panels being produced for that segment will help push down prices for LCD TVs,analysts predicted( Sharp Electronics,for one,is betting heavily on LCDs(Its chairman,Toshiaki Urushisako, has predicted that Sharp will switch completely from conventional tube sets to LCD TVs in Japan by 2005( Flat-panel refers to wafer-thin(3 inches or less)TVs,whereas flat-screen may actually describe traditional cathode-ray-tube sets(CRTs)whose glass front lacks the distorting curve that TVs have had for 50 years( Be aware of two things:One,flat-panel technology may not be high-definition TV; for eventual HDTV reception,some of these sets will require a separate HD tuner(Two,some flat-panel TVs are just the panel and lack speakers and sometimes a built-in tuner( Price range:US ,700,2,000 LCD vs plasma In general,LCD technology is used for smaller screens because of the enormous number of transistors needed to turn the glasslike liquid crystals into color images(The larger the display,the more transistors,the more chance of failed connections( A plasma screen is found in TV sets larger than 20 inches(Color is comparable to an LCD’s(LCDs do not deteriorate over time,while a plasma display averages 30,000 hours(a traditional TV screen can go for 20,000),after which it fades over a period of years(Earlier problems with the quality of plasma’s contrast have been addressed, and current screens are cleaner and better defined( Price range:US ,600,2,800 HDTV Simply put,high-definition TV is 10 times as sharp as traditional TV,and the sound is digital,like CD sound,not FM ,which is what traditional TV provides(HD technology achieves its visual clarity with more immage lines on the screen(Where analog TVs have 480 horizontal lines,HDTV has 720 or 1,080lines(Be aware : m any HDTV sets being sold now are in fact only HDTV monitors ,offering a crisp picture (To receive genuine high-definition television signals,owners must buy a separate HDTV receiver( Price range:US ,1,000,6,000 LOS ANGELEST IMES 81(According to the article,TV sets with _____will still be the most popular in the coming years( 82(The factors that stimulate the thin screens to be more and more popular include _____,_____,and_____( 第 27 页 共 77 页 2003-2010年全国大学生英语竞赛C类初赛、决赛真题及答案汇总集(原创) 83(The rapidly expanding market for LCD panels and their large-scale production will help lower _____( 84(When a wealthy customer wants to buy a very large TV,he should select _____and _____according to the passage( 85(We can infer from the passage that among all kinds of TV sets _____is of the best quality( Questions 86 to 90 are based on the following passage: Dell Does Domination Over the years I’ve spent a fair amount of time hanging out with Michael Dell,and what I noticed during my latest visit with him in Austin is how things have changed(Yes,he is still unflappable(And yes,he greets me in his new glossy offices with the same Stamford Wife-like grin he has always had(But he appears thinner now,as if he’s lost baby fat(While he’s still slow-moving,as if he’s conserving energy,he now cuts to the quick in conversation(And when he zeroes in on the point he wants to make,when he reiterates why Dell Computer is in a better position than any other PC maker in the world,you realize that the 36-year-old has lost what was once one of his greatest advantages:no one underestimate ates him anymore( Instead,Michael Dell looms over the PC landscape like a giant,casting a shadow over all his unfortunate com petitors(This is a terrible time in a difficult business(PC sales were down for the first time last year(Dell’s sales will be down,too,also for the first time(Yet even with that,even with recession ,even with the threat of a Hewlett-Packard ,Com paq Goliath,this is the only PC maker you can count on to grow and grow and grow(Almost single-handedly,Dell is forcing this industry to consolidate(Could this mean“game over”in the PC biz,“Game over,”he looks back at me incredulously(“No way(We only have 14, global market share(” The Dellites may not admit to“game over”aspirations,but clearly they are thinking of a kind of domination never seen before among PC makers(“We think 40,market share is possible,”says Dell’s No(2,Kevin Rollins(That’s a remarkable goal;what’s more remarkable is that it really is attainable(Don’t look for Dell to hit that kind of number anytime soon(Rather,the company’s growth will come from grinding out gains on several existing fronts,while shrewdly expanding into new target markets( The reason is simple:there’s no better way to make,sell,and deliver PCs than the way Dell does it,and nobody executes that model better than Dell(By now most business people can recite the basic tenets of Dell’s direct-sales model(Dell machines are made to order and delivered directly to the customer(There is no middleman(The customer gets the exact machine he wants cheaper than he can get it from the competition(The company gets paid by the customer weeks before it pays suppliers(Given all that,the company that famously started in Austin out of a University of Texas dorm room now dominates the northern side of this city the way giant steelworks once lorded over old mill towns(Dell has some 24 facilities in and near Austin and employs more than 18,000 local workers(Dell did over ,30billion in sales in 2000,ranking 48th on the FORTUNE 500,ahead of names like Walt Disney,and Du Pont(Michael is the richest man under 40 in the world,worth ,16 billion( Two facts show how well the Dell model is working,even in tough times:Dell is on track to earn over ,1.7 billion in 2001,taking almost every single dollar of profit among makers of Windows-based PCs(And Dell is gaining market share(That’s not true for any other major PC maker( Quite the contrary(The others are going splat for the same reason that Dell is succeeding: 第 28 页 共 77 页 2003-2010年全国大学生英语竞赛C类初赛、决赛真题及答案汇总集(原创) commoditization(The desktop PC has become a commodity(That’s great for consumers,who get standardized,easy-to-use,cheap PCs( Commoditization has been going on in the industry for years(Dell,as master of the direct model,spent most of the 1990s operating in techno-Nirvana(The PC market was growing by 15,-plus per year(For its quarter ended January 2000,Dell did a record ,6(8 billion in sales,up 31, from the previous year’s quarter(In a sign of things to come,sales growth slowed later in 2000(Then the growth disappeared in 2001( The economic slowdown was bad news for everyone,but Michael Dell and Kevin Rollins,who is increasingly his equal partner in running this business,made sure it was terrible news for Dell’s com petitors(In late 2000they decided to slash prices(“It was advantageous for us,actually,because in periods of slow demand component prices drop,and, unlike our competition,we can pass those savings on immediately to customers,”explains Rollins,a fine violinist who grew up in a hard-charging Utah family—his father was an engineering professor at Brigham Young—and came to Dell from the Bain consulting firm (Dell could make more money selling more computers at lower prices than it could selling fewer computers at higher prices(The low prices wreaked havoc on competitors(Compaq,HP,and Gateway all lost market share for the 12 months that ended Sept(30,2001,while Dell’s share of the U(S(market climbed 31,( 86(_____was once one of Dell’s greatest advantages( 87(The passage attributes the success of Dell to_____( 88(Why did Dell’s share of the U.S(market climb 31, in 2001 while the economy slowed down, 89(Dell hopes to increase its global market share by _____according to the passage( 90(If one wants to buy a Dell computer,who do you think he is supposed to pay, Part V Word Guessing and IQ Test(5 minutes,10 points) Section A Word Guessing(5 points) 91(In the western countries,a person who serves the guests in a restaurant expects a gratuity in appreciation for good service( A(good remark B(letter of thanks C(tip D(promotion 92(In Greek mythology,the hero usually finds himself in a terrible predicament in which he has to face a fate which has been foretold by the Oracle at Delphi( A(strange phenomenon B(unpleasant situation C(unusual circumstance D(huge monster 93(A first-class business establishment will fully recompense its customers if there is ever any dissatisfaction with the service or product( A(compensate B(understand C(inform D(award 94(Our family was truly a matriarchy,because my grandmother had to raise her children and provide for them all by herself;my grandfather had died when his sons and daughter were quite young( A(family governed by women B(family without the father C(family depending on the mother D(family with many young children 95(I’d love to help you look for all those things but I’m a bit pushed for time today(Why don’t we split up and meet back in a couple of hours for a quick cuppa and then I can have a look at what you’ve bought( A(put it aside B(go separately 第 29 页 共 77 页 2003-2010年全国大学生英语竞赛C类初赛、决赛真题及答案汇总集(原创) C(save the time D(do it one by one Section B IQ Test(5 points) 96(What number should replace the question mark, A(10 B(9 C(8 D(7 97(If someone could take a sheet of paper 0(1mm thick,tear it in half,put the pieces on top of each other,tear these in half,then repeat the process until the paper had been torn 25 times,what would be the final height of the pile of paper, A(As thick as a book( B(As tall as a man( C(As high as a house( D(As high as a mountain( 98(If the pyramid were flattened out would it look like A,B,C or D, 99(Only one of the sets of six letter below can be rearranged into a six-letter word in the English language(Can you find that set, A(LO RIDM B(ETNMIU C(TUBLID D(MIRCEL 100(These are millennium celebrations taking place in 4 different places(Which one occurred first, A. PARIS B. SYDNEY C. NEW YORK D. GREAT WALL OF CHINA Part VI Translation(10 minutes,10 points) Directions:Translate the underlined sentences of the following passage into Chinese on the Answer Sheet( TASTE A WORLD OF DIFFERENCE Wherever you are in the world,it seems,the billboards and supermarket shelves are saturated with the same brand names(There are few places left on the planet where you can’t buy a Diet Coke,a packet of Kellogg’s cornflakes,or Oxo cubes( But just try tasting them((101)The packaging and the product may look identical, but the flavour maybe far from familiar(Even the most famous brand names are specially for-mulated to appeal to individual national palates(Heinz,for instance, insists that its tomato ketchup is“the same recipe worldwide”,but admits that“there maybe very subtle variations in the spicing”( Even the ultimate global brand,Coca-Cola,accepts that not all its products are what they appear to be(“We go to great lengths to ensure that Coca-Cola is the same wherever you drink it,”says a spokesman(“But Diet Coke may change slightly from country to country,because we use different sweeteners in different places(” Americans prefer many products,particularly chocolate,to be far more sugary than Europeans do((102)The French perceive strawberry flavour in a different way from the British,so the artificial flavouring in France will not have a“proper”strawberry taste to Britons(Preferences for saltiness and color differ as well,and most Japanese consumers dislike the taste of milk( The French in general prefer stronger,fuller-tasting coffee than the British(There is a higher cream content in a Magnum in Italy than in Britain,because the Italians expect ice-cream to have a much richer taste(Tea sold in Europe tends to be weaker than the strong tea preferred by Britons((103)Taste preferences can vary even within countries:at least one well-known soft 第 30 页 共 77 页 2003-2010年全国大学生英语竞赛C类初赛、决赛真题及答案汇总集(原创) drink is made according to a sweeter recipe in the southern states of the United States than that on sale in the north( Local water and soil will affect the taste of home-grown natural ingredients(Some key constituents may not be available,meaning that alternatives must be found(Sometimes,especially in developing countries,premium grade components, such as high-grade flour,are replaced with lower-quality equivalents( (104)Extremes in climate will require different additives and preservatives to be used( Chocolate calls for an alternative recipe in hot countries if it is to maintain its texture and taste( And multinationals must conform to national laws and regulations on additives, flavorings,colorings,and artificial low-calorie sweeteners(Mars and Snickers bars taste slightly different in Australia,since laws there state that vegetable fat must not be used in chocolate( Local religious sensibilities must also be observed((105)Even McDonald’s,which proclaims the homogeneity of its Big Macs(巨无霸)around the globe,has to serve lamb rather than beef burgers in India,because the sacred status of cows means that the majority of its Indian customers can’t eat beef( Part VII Writing(30 minutes,20 points) Directions:The chart below shows the amount of leisure time enjoyed by men and women of different employment status(Write a report for a university lecturer describing the information shown below(You should write at least 150 words( Leisure time in a typical week:by sex and employment status,1998,99 2003年全国大学生英语竞赛初赛赛卷答案 Part I Listening Comprehension (30 分) Section A Dialogues (10 分, 每题1分) 1. C. 2. C 3. C 4. B 5. D 6. C 7. A 8. A 9. B 10. B Section B News Items (10 分, 每题1分) 11. The UN Security Council’s resolution. 12. $100 million. 13. About a deadly shooting. 14. 14. 15. A way to reduce early births among women at the risk of premature delivery. 16. The Bush administration. 17. A fire in a night club. 18. To seek UN approval for war. 19. Murder and injury. 20. A gunshot. Section C Compound Dictation (10 分,21—28题,每题0.5分;29、30题, 每题3分) 21. purchasing 22. firms 23. engage 24. factors 25. concerned 26. climate 27. introducing 28. characters 29. This critical problem of proper translating is only one of many cultural differences facing American corporations overseas 30. Because of their cultural heritage, businesspersons in each country conduct their activities 第 31 页 共 77 页 2003-2010年全国大学生英语竞赛C类初赛、决赛真题及答案汇总集(原创) differently. Part II Vocabulary and Structure (30 分) Section A Multiple Choice (20 分, 每题1分) 31. B 32. D 33. A 34. C 35. C 36. A 37. D 38. B 39. B 40. A 41. D 42. C 43. B 44. C 45. B 46. B 47. A 48. D 49. D 50. C Section B Error Correction (10 分, 每题1分) 51. ? 52. ago? later 53. interlocked ? interlocking 54. free?? them 55. weighing? weigh 56. twenty ? twentieth 57. mind ? remind 58. caught ? catch 59. luggages ? luggage 60. a ? 删除a Part III Situational Dialogues (10 分, 每题1分) 61. D 62. A 63. C 64. B 65. B 66. B 67. A 68. C 69. D 70. D Part IV Reading Comprehension ( 40 分) Section A Multiple Choice (10分, 每题2分) 71. A 72. C 73. B 74. A 75. B Section B Short Answer Questions (30分, 每题2分) 76. increase your social inventiveness 77. theoretical/philosophical 78. determined 79. give it a rest 80. get them into the world / make them known 81. bulky picture tubes 82. price cuts, convenience, the demands for high definition 83. prices for LCD TVs 84. LCD, HDTV 85. HDTV 86. That he was once underestimated / That someone once underestimated him 87. The direct sales model / commoditization 88. They slashed the prices. / They lowered the prices. 89. 26% 90. The Dell company. Part V Word Guessing and IQ Test (10分) Section A Word Guessing (5 分,每题1分) 91. C 92. B 93. A 94. A 95. B Section B IQ Test (5 分,每题1分) 96. C 97. D 98. A 99. B 100. B Part VI Translation (10 points) 101. 包装和产品也许看起来是完全一样的,但是味道也许跟所熟悉的相差甚 远。 102. 法国人对草莓口味的感觉与英国人不同,所以法国产的草莓人工调味剂不 合英国人的胃口。 103. 甚至在一国之内,口味偏好也会有极大差别:至少一种指明软饮料,在美 国南部地区是用比在北方销售的饮料甜味更重的配方制成的。 104. 气候的巨大差别要求使用不同的添加剂和防腐剂。 105. 即使宣称在全球销售同一种类的巨无霸的麦当劳也不得不在印度提供羊 肉汉堡而不是牛肉汉堡,原因是牛的神圣地位意味着大多数印度顾客不能食用牛 肉。 第 32 页 共 77 页 2003-2010年全国大学生英语竞赛C类初赛、决赛真题及答案汇总集(原创) Part VII Writing (20分) (参考范文) The chart shows the number of hours of leisure enjoyed by men and women in a typical week in 1998-99, according to gender and employment status. Among those employed full-time, men on average had forty-seven hours of leisure, whereas women had approximately thirty-seven hours. There were no figures given for male part-time workers, but female part-timers had thirty-nine hours of leisure time, only slightly more than women in full-time employment, perhaps reflecting their work in the home. In the unemployed and retired categories, leisure time showed an increase for both sexes, as might have been expected. Here too, men enjoyed more leisure time---over eighty hours, compared with less than seventy hours for women, perhaps once again reflecting the fact women spend more time working in the home than men. Lastly, housewives enjoyed approximately fifty-four hours of leisure, on average. There were no figures given for househusbands! Overall, the chart demonstrates that in the categories for which statistics on male leisure time were available, men enjoyed at least ten hours of extra time. 2004年全国大学生英语竞赛初赛试题 2004 National English Contest for College Students (Preliminary) Part I Listening Comprehension (30 minutes, 30 points) Section A Dialogues (10 points) Directions: In this section, you will hear 10 short dialogues. At the end of each dialogue, a question will be asked about what was said. Both the dialogue and the question will be read only once. After each question ,there will be a pause. During the pause, you must read the four choices marked A, B, C and D, and decide which is the best answer. Then mark the corresponding letter on the Answer Sheet with a single line through the centre. 1. A. In San Francisco. B. At an airport. C. At a travel agency. D. In a post office. 2. A. The woman is going out to lunch. B. The woman wants to eat some chocolate. C. The woman will go to a convenience store. D. The woman will be back in 30 minutes. 3. A. By car. B. By plane. C. By train. D. By ferry. 4. A. She had lost her job. B. She didn’t know the mayor. C. She was mistaken. D. The man misunderstood her. 5. A. He needs some tomato juice. B. His shirt is stained. C. He needs his shirt by tomorrow. D. His shirt is missing. 6. A. To a meeting. B. To the office. C. To a restaurant. D. To a bowling class. 7. A. Give the woman some medicine. 第 33 页 共 77 页 2003-2010年全国大学生英语竞赛C类初赛、决赛真题及答案汇总集(原创) B. Find out more about the woman’s injury. C. Test the strength of the woman’s shoulder. D. Go skiing with the woman. 8. A. Excited. B. Thankful. C. Somewhat disappointed. D. Somewhat bothered. 9. A. When her family celebration is over. B. After the man graduates from school. C. After they have some pictures taken together. D. When she has bought her cap and gown. 10. A. By continuous assessment. B. By giving a per cent. C. By giving grade. D. By means of exams. Section B News Items (10 points) Directions: In this section, you will hear 10 short pieces of news from BBC or VOA. After each news item and question,there will be a pause. During the pause, you must read the three choices marked A, B and C, and decide which is the best answer. Then mark the corresponding letter on the Answer Sheet with a single line through the centre. 11. A. Under the age of four. B. Under the age of five. C. Under the age of six. 12. A. Beijing’s successful bid for the 2008 Olympic Games. B. The Organizing Committee of the 2008 Olympic Games. C. The large market of the Olymic brand. 13. A. To keep the code for its Windows operating system a secret. B. To design some new computer software. C. To persuade more PC users to adopt the Windows operating system. 14. A. One. B. Ten. C. Thirty. 15. A. No. B. Yes. C. Not mentioned. 16. A. More than 500 dollars. B. A little more than three dollars. C. Less than three dollars. 17. A. Because the Iraqi economy has gradually risen after the war. B. Because Iraqi people trust the new dinar more. C. Both A and B. 18. A. Five. B. Six. C. Seven. 19. A. Low fruit and vegetable intake. B. Smoking and little exercise. C. Unhealthy diet. 20. A. The euro has risen in value. B. The US dollar has risen in value. C. German economy has slided into recession. 第 34 页 共 77 页 2003-2010年全国大学生英语竞赛C类初赛、决赛真题及答案汇总集(原创) Section C Passages (10 points) Directions:In this section, you will hear 2 passages. At the end of each passage, you will hear 5 questions. After you hear a question, you must choose the best answer from the four choices marked A, B, C and D. Then mark the corresponding letter on the Answer Sheet with a single line through the centre. Passage One 21. A. Jazz. B. Indian. C. Country. D. Pop. 22. A. Their hairstyles. B. Their humor. C. Their clothing. D. All of the above. 23. A. America. B. England. C. Italy. D. Canada. 24. A. The Beatles were formed in England. B. The Beatles had a successful movie career. C. The Beatles are regarded as one of the finest jazz groups. D. The Beatles first recorded music in 1962. 25. A. Seventeen. B. Twenty-eight. C. Twenty-two. D. Twelve. Passage Two 26. A. Because he regarded the bear as his friend. B. Because the bear was beautiful. C. Because he considered it poor sportsmanship to shoot a tied-up animal. D. Because bears are not dangerous animals. 27. A. Because Teddy is the nickname for Theodore Roosevelt. B. Because it was then the usual practice to do so. C. Because it was the first time to name toy bears Teddy Bears. D. Because the toy bear was made to look a bit like the President. 28. A. People collect Teddy Bears. B. Teddy Bears can be found in museums. C. The first Teddy Bear was made by Mr. Mitchtom’s wife. D. President Roosevelt shot the black bear in 1902. 29. A. Seven. B. Six. C. Two. D. Five. 30. A. He moved to Florida. B. He became President. C. He drew cartoons. D. He started a toy company. Part II Vocabulary and Structure (10 minutes, 20 points) Section A Multiple Choice (10 points) Directions:There are 7 incomplete sentences and 3 incomplete dialogues in this section. For each blank there are four choices marked A, B, C and D. Choose the one that best completes the sentences and dialogues. Then mark the corresponding letter on the Answer Sheet with a single line through the centre. 31. Never________the power of your actions. With one small gesture you can change a person’s life. A. underestimate B. overvalue 第 35 页 共 77 页 2003-2010年全国大学生英语竞赛C类初赛、决赛真题及答案汇总集(原创) C. misuse D. dismiss 32. Scientists have warned that penguins in the Antarctic could be very________to changes in climate and could be threatened by any long-term temperature shifts. A. superstitious B. acceptable C. suspicious D. susceptible 33. Since settling in Scotland I ________ golf as a hobby. A. have taken up B. took up C. have taken in D. took in 34. She often thinks that her six years in Italy were wasted, ________she________ that time learning more Italian. A. but that; might have taken B. for that; should have found C. in that; could have spent D. with that; would have used 35. He constantly________his proposal that________of the budget surplus be used to offer a voluntary prescription drug benefit to seniors. A. views; many B. reiterates; a part C. complains; a great amount D. thinks; lots 36. ________Alan’s amazement, the passport office was closed when he arrived. A. With B. For C. To D. Of 37. I was asked the other day whether high and low pressure systems were________the central pressure. A. maintained to B. determined by C. generated within D. preserved to 38. Bob: What are you reading, Frank? Tom: It’s this week’s New Scientist, why? Bob: I was just wondering—________, but I’ve never actually read it myself. Is it aimed at real scientists or can ordinary people like me understand it? A. it’s for anyone really B. where I can buy it C. it seems very expensive D. it looks interesting 39. Girl: Hi Paul—looking forward to your holiday? Boy: Oh, yeah—it’s going to be great. Though I’m a bit worried that I’ve packed the wrong clothes. I don’t think the weather’s going to be as good as I hoped. Girl:________ Boy: That’s right—my first flight. A. Everything will be OK, isn’t it? B. You’re flying on Saturday, aren’t you? C. It’s far from here, as everybody knows. D. That’s a good idea, anyway. 40. John: What plastic products do you have in mind that are easy to recycle? Tom: Shampoo bottles, detergent bottles, medicine bottles, food containers, etc. They are all easily collectable and reusable. John: ________, but actually I think you are missing the point of recycling. It doesn’t just mean using old bottles again and again for the same purpose. What it means these days is melting the 第 36 页 共 77 页 2003-2010年全国大学生英语竞赛C类初赛、决赛真题及答案汇总集(原创) plastics down and building them up again into some completely new product. A. Not too bad B. Something is wrong C. You’re right there D. It’s a new idea Section B Cloze-Test (10 points) Directions: There are 10 blanks in the passage. For each blank there are four choices marked A, B, C and D. Choose the one that best completes the sentence. Then mark the corresponding letter on the Answer Sheet with a single line through the centre. Ask most people for their list of Top Ten fears, and you’ll be sure to find being burgled fairly high on the list. An informal survey I carried out among friends at a party last week (41)________that eight of them had their homes broken into more than twice, and two had been burgled five times.To put the record (42)________, none of my friends owns valuable paintings or a sideboard full of family silverware. Three of them are students, in fact. The most (43)________burglary, it seems, involves the theft of easily transportable items—the television, the video, even food from the freezer. This may have something to do with the fact that the average burglar is(44)________his (or her) late teens, and probably wouldn’t know what to do with a Picasso, (45)________selling a Walkman or a vacuum cleaner is a much easier matter. They are perhaps not so much (46)________criminals as hard-up young people who need a few pounds and some excitement. (47)________that this makes having your house turned upside down and your favourite things stolen any easier to accept. In most cases, the police have no luck (48)________any of the stolen goods. Unless there is any (49)________evidence, they are probably unable to do anything at all. And alarms or special locks don’t seem to help either. The only advice my friends could (50)________up with was “Never live on the ground floor” and “Keep two or three very fierce dogs”. 41. A. released B. revealed C. reclaimed D. redeemed 42. A. straight B. clear C. apparent D. correct 43. A. typical B. abnormal C. hazardous D. vicious 44. A. near B. in C. beyond D. out of 45. A. whereas B. whenever C. however D. once 46. A. serious B. professional C. efficient D. perfect 47. A. Given B. Even C. Not D. Despite 48. A. seizing B. withdrawing C. seeking D. recovering 49. A. distinguishable B. obscure C. outstanding D. definite 50. A. come B. catch C. keep D. put Part III Word Guessing and IQ Test (5 minutes, 10 points) Section A Word Guessing (5 points) 51. Social capital has become a mantra for politicians and policy makers: they see it as a bulwark against society’s ills and a means of multiplying the effects of financial investment in social projects. A. measurement B. defense C. treatment D. complaint 52. Her desire for anonymity soon became apparent when she refused to answer questions about her identity. A. recognition B. concealment C. vanity D. success 第 37 页 共 77 页 2003-2010年全国大学生英语竞赛C类初赛、决赛真题及答案汇总集(原创) 53. “Gentlemen,” replied Candide, with a most engaging modesty, “you do me much honor, but upon my word I have no money.” A. you help me a lot B. you are great C. it’s very generous of you to say so D. that’s a great honor for me 54. When it comes to listening to the opinions of members of your school community, do you think you are already “all ears?” A. in full strength B. bearing ideas in mind C. ready to listen attentively D. having enough preparation 55. That extremely indolent student will clean out his desk when pigs fly or I am much mistaken as to his character. A. soon B. never C. sometime D. often Section B IQ Test (5 points) 56. Sally had a third again as many as David, who had a third as many again as Francis. Altogether they had 111. How many did David have? A. 27 B. 32 C. 36 D. 48 57. What letter should replace the question mark? A. T B. S C. I D. N 58. BONA FIDE is to genuine as DE FACTO is to________. A. together B. actual C. reason D. assumed 59. How many revolutions must the largest cog make in order to bring the cogs back to their original positions? A. 56 B. 48 C. 36 D. 12 60. The diagram shows a small village church. There is a door in the west end, seen in the diagram. There is a tower at the east end of the church with a window set in its east wall. This wall is hidden in the diagram.There is also a door in the tower. Which of these is most likely to be the view of the eastern end of the church? Part IV Reading Comprehension (25 minutes,30 points) Directions:In this part there are 5 passages with 30 questions or incomplete statements. Read the passages carefully. Then answer the questions in the fewest possible words(not exceeding 10 words). Remember to rewrite the answers on the Answer Sheet. Questions 61 to 66 are based on the following passage: Centuries ago, man discovered that removing moisture from food helps to preserve it, and that the easiest way to do this is to expose the food to sun and wind. Fruit is sun-dried in Asia Minor, Greece, Spain and other Mediterranean countries, and also in California, South Africa and Australia. The methods used vary, but in general, the fruit is spread 第 38 页 共 77 页 2003-2010年全国大学生英语竞赛C类初赛、决赛真题及答案汇总集(原创) out on trays in drying yards in the hot sun. In order to prevent darkening, pears, peaches and apricots are exposed to the fumes of burning sulphur before drying. Plums, for making prunes, and certain varieties of grapes for making raisins and currants, are dipped in an alkaline solution in order to crack the skins of the fruit slightly and remove their wax coating, so increasing the rate ofdrying. Nowadays most foods are dried mechanically. The conventional method of such dehydration is to put food in chambers through which hot air is blown at temperatures of about 110? at entry to about 43? at exit. This is the usual method for drying such things as vegetables, minced meat, and fish. Liquids such as milk, coffee, tea, soups and eggs may be dried by pouring them over a heated horizontal steel cylinder or by spraying them into a chamber through which a current of hot air passes. In the first case, the dried material is scraped off the roller as a thin film which is then broken up into small, though still relatively coarse flakes. In the second process it falls to the bottom of the chamber as a fine powder. Where recognizable pieces of meat and vegetables are required, as in soup, the ingredients are dried separately and then mixed. Dried foods take up less room and weigh less than the same food packed in cans or frozen, and they do not need to be stored in special conditions. For these reasons they are invaluable to climbers, explorers and soldiers in battle, who have little storage space. They are also popular with housewives because it takes so little time to cook them. Usually it is just a case of replacing the dried-out moisture with boiling water. Questions: 61. Fruit is sun-dried generally on________. 62. Why are sulphur fumes used before drying some fruits? 63. Where are vegetables commonly dried nowadays? 64. If soup requires recognizable pieces of meat, they are________. 65. Dried foods are often used by________, ________and________. 66. Why do housewives like dried foods? Questions 67 to 72 are based on the following passage: Hollywood writers honor Coppola, “Splendor” Sunday, February 22, 2004 Posted: 9:56 AM EST (14:56 GMT) LOS ANGELES, California (Reuters) —Hollywood’s screenwriters Saturday snubbed the final installment of the highly acclaimed Lord of the Rings trilogy and instead awarded a key prize to a low-budget film based on a comic book writer. American Splendor, which revolves around the travails of comics connoisseur Harvey Pekar, won the Writers Guild of America Award for best adapted screenplay, while writer / director Sofia Coppola’s Lost in Translation, about a pair of mismatched Americans languishing in Tokyo, nabbed the trophy for original screenplay. The American Splendor screenplay was written by the film’s directors, Robert Springer and Shari Springer Berman, who were not present at the awards. The Writers Guild of America Awards were handed out simultaneously in Los Angeles and New York. Coppola said she was excited to be honored by the union. “I find it difficult to write, so it’s very encouraging and exciting to get an award,” Coppola told Reuters after the event. Coppola’s competition was Gurinder Chadha, Paul Mayeda Berges and Guljit Bindra for Bend It 第 39 页 共 77 页 2003-2010年全国大学生英语竞赛C类初赛、决赛真题及答案汇总集(原创) Like Beckham, Steven Knight for Dirty Pretty Things, Irish director Jim Sheridan and his daughters Naomi and Kirsten for In America, and first-time writer / director Tom McCarthy for The Station Agent. The other adapted screenplay nominees were director Peter Jackson, Fran Walsh and Philippa Boyens for The Lord of the Rings: The Return of the King, Anthony Minghella for Cold Mountain, Brian Helgeland for Mystic River, and writer / director Gary Ross for Seabiscuit. Coppola, Knight and the Sheridans will vie for the Academy Award next week, along with the writers of The Barbarian Invasions and Finding Nemo. Apart from Cold Mountain, all the Writers Guild of America adapted screenplay contenders will compete for the Oscar, along with the Brazilian drama City of God. In the last 12 years, eight of the Writers Guild of America adapted screenplay winners and seven of its original screenplay winners have gone on to Oscar glory. Lost in Translation has already picked up three Golden Globes—an Oscar bell-wether—including best screenplay. The Lord of the Rings: The Return of the King, which has 11 Oscar nominations, has collected four Golden Globes and prizes from Hollywood’s producers and directors guilds. Questions: 67. What does American Splendor mainly write about? 68. Sofia Coppola’s Lost in Translation won the Writers Guild of America Award for________. 69. The Writers Guild of America Awards were offered at the same time in________and________. 70. How did Coppola feel about her getting the award? 71. List at least 3 movies that competed with Lost in Translation. 72. How many Writers Guild of America screenplay winners have gone on to Oscar glory in the last 12 years? Questions 73 to 78 are based on the following passage: The need for a surgical operation, especially an emergency operation, almost always comes as a severe shock to the patient and his family. Despite modern advances, most people still have an irrational fear of hospitals and anaesthetics. In the early years of last century there was little specialization in surgery. A good surgeon was capable of performing almost every operation that had been devised up to that time. Today the situation is different. Operations are now being carried out that were not even dreamed of fifty years ago. The heart can be safely opened and its valves repaired. Clogged blood vessels can be cleaned out, and broken ones mended or replaced. A lung, the whole stomach, or even part of the brain can be removed and still permit the patient to live a comfortable and satisfactory life. The scope of surgery has increased remarkably in 20th century. Its safety has increased too. Deaths from most operations are about 20% of what they were in 1910 and surgery has been extended in many directions,for example to certain types of birth defects in newborn babies, and, at the other end of the scale, to life-saving operations for the octogenarian.The hospital stay after surgery has been shortened to as little as a week for most major operations. Many developments in modern surgery are almost incredible. They include the replacement of damaged blood vessels with simulated ones made of plastic; the replacement of heart valves with plastic substitutes; the transplanting of tissues such as the lens of the eye. One of the most revolutionary areas of modern surgery is that of organ transplants. Until a few years ago, no person, except an indentical twin, was able to accept into his body the tissues of another person without reacting against them and eventually causing death. Recently, however, it 第 40 页 共 77 页 2003-2010年全国大学生英语竞赛C类初赛、决赛真题及答案汇总集(原创) has been discovered that with the use of x-rays and special drugs, it is possible to graft tissues from one person to another which will survive for periods of a year or more. Kidneys have been successfully transplanted between non-identical twins. Heart and lung transplants have been reasonably successful in animals, though rejection problems in humans have yet to be solved. “Spare parts” surgery, the simple routine replacement of all worn-out organs by new ones, is still a dream of the distant future. As yet, surgery is not ready for such miracles. In the meantime, you can be happy if your doctor says to you,“Yes, I think it is possible to operate on you for this condition.” Questions: 73. Most people are afraid of being operated on in spite of________. 74.A patient can still live a comfortable and satisfactory life even after the removal of________. 75. Today deaths from most operations are about ________of what they were in 1910. 76. What’s the main difficulty in organ transplanting? 77. Is “spare parts” surgery possible now? 78. You can be happy if your surgeon can operate because it means________. Questions 79 to 84 are based on the following passage: Sales of anti-ageing skin treatments have reached a new high as British women try to stay younger longer (writes Joanna Bale). But according to a recent survey of those aged between 35 and 55 there are significant regional variations in annual spend on these cosmetics. While the average woman thinks costs of ,200 a year acceptable—almost treble the ,75 of three years ago—some fork out ,500, according to the survey of over 2,000 women nationwide by the beauty company Olay. Brows will wrinkle at the differences in yearly spend on anti-ageing treatments across major cities: the ladies of Edinburgh spent least, just ,50 a year, while those in Leeds spend most, at a staggering ,500. London women most commonly spend ,200 annually, and those in Manchester give themselves a modest yearly budget of ,100. Equally surprising are the results among 40-somethings. Single women spend the least on indulging their desire for facial rejuvenation, with only 25 per cent forking out on skincare products or treatments. This figure rises to just over 31 per cent of married women and a similar figure for those who are unmarried with partners. By far the biggest spenders are the 50 per cent of divorcees who feel the pressure to invest heavily in their facial futures. The survey highlighted two groups who spend significant sums on enhancing their appearances—“Sindies” (single income now divorced), and women in their 40s who use their looks to get ahead. The survey also found that although women wanted to “de-age” they had a holistic view of beauty and embraced a realistic and natural approach to looking good. Questions: 79. Compared with that of three years ago, the average British woman’s annual spend on cosmetics has almost________. 80. Women in________spend most on cosmetics according to the survey. 81. The ladies of Birmingham as well as those in ________spend ,100 a year on antiageing treatments. 第 41 页 共 77 页 2003-2010年全国大学生英语竞赛C类初赛、决赛真题及答案汇总集(原创) 82. ________and________spend more on indulging their desire for facial rejuvenation than single women. 83. Enhancing appearances plays an important role in the life of________and________. 84. After reading the passage, what do you learn about the sales of anti-ageing skin treatments now in Britain? Questions 85 to 90 are based on the following passage: It is hard to get any agreement on the precise meaning of the term “social class”. In everyday life, people tend to have a different approach to those they consider higher or lower than themselves in the social scale. The criteria we use to “place” a new acquaintance, however, are a complex mixture of factors. Dress, way of speaking, area of residence in a given city or province, education and manners all play a part. In ancient civilizations, the Sumerian, for example,social differences were based on birth,status or rank,rather than on wealth. Four main classes were recognized. These were the rulers, the priestly administrators, the freemen (such as craftsmen,merchants or farmers) and the slaves. In Greece, after the sixth-century B.C., there was a growing conflict between the peasants and the landed aristocrats, and a gradual decrease in the power of the aristocracy when a kind of “middle class” of traders and skilled workers grew up. The population of Athens,for example, was divided into three main classes which were politically and legally distinct. About one-third of the total were slaves, who did not count politically at all, a fact often forgotten by those who praise Athens as the nursery of democracy. The next main group consisted of resident foreigners, the “metics”, who were freemen, though they too were allowed no share in political life. The third group was the powerful body of “citizens”, who were themselves divided into subclasses. In ancient Rome, too, a similar struggle between the plebs, or working people, and the landed families was a recurrent feature of social life. The medieval feudal system, which flourished in Europe from the ninth to the thirteenth centuries, gave rise to a comparatively simple system based on birth.Under the king there were two main classes—lords and “vassals”, the latter with many subdivisions. The vassal owed the lord fidelity, obedience and aid, especially in the form of military service. The lord in return owed his vassal protection and an assured livelihood. In the later Middle Ages, however, the development of a money economy and the growth of cities and trade led to the rise of another class, the “burghers” or city merchants and mayors. These were the predecessors of the modern middle classes. Gradually high office and occupation assumed importance in determining social position, as it became more and more possible for a person born to one station in life to move to another. This change affected the towns more than the country areas, where remnants of feudalism lasted much longer. Questions: 85. List at least three common criteria for telling a person’s social position. 86. What were the four main classes in the Sumerian civilization? 87. Slaves in Greece in the sixth century B.C. were not________significant. 88. The struggle between the plebs and the landed families was a________feature of social life. 89. The metics,one of the three classes of Greece, consisted mainly of________. 90. What did the development of a money economyand the growth of cities and trade lead to? Part V Error Correction (5 minutes,10 points) Directions:The following passage contains 9 errors. In each case only one word is involved. You 第 42 页 共 77 页 2003-2010年全国大学生英语竞赛C类初赛、决赛真题及答案汇总集(原创) should proofread the passage on the Answer Sheet and correct it in the following way: EXAMPLE One night,quite late,I was still awake in the room I am shared with 1. am my husband. I was lying on my right side and can hear a child crying. 2. could Getting up,I went ? see if our son was all right. 3. to He was sleeping soundly,breathing deeply and gently. 4. ? If the air in New York seems a little less grimy this spring, thank Rudolph Giuliani. On January 10th, after months of burning debate, the city’s non-smoke mayor 91.________ signed the Smoke-Free Air Act. From April 10th smoking will be stubbed out(碾灭) in restaurants catering for more than 35 people, a move that will hit about half the city’s 11,000 eating places. Nicotine addicts will also smoked out at work, except 92.________ in ventilated smoking rooms or offices occupied by no more than three consenting adults. More radically, outdoor seating areas will also become smoke zones. 93.________ Come the new baseball season, fans at Yankee Stadium will be breaking the law if they light up. New York joins well over 100 American cities—and four states—that have passed laws banned smoking 94.________ in public places. More than a third of American companies now forbid smoking in the workplace, up to 95.________ a mere 20% in 1986. And the tobacco industry, which in America alone has annual sales of close to ,50 billion, is watching its profits go down in smoke. 96.________ The industry may never recover. Polls suggest that nine out of ten Americans are irritated by cigarette smoke. With good reason. In 1993 the Environmental Protection Agency (EPA) has classified 97.________ “second-hand” smoke as a health hazard—one that,according to the EPA, causes 3,000 non-smokers to die from lung cancer each year. 98.________ New Yorkers must now wait and see if the pro-smoking lobby’s alarming predictions of citywide economical collapse come true. Tobacco 99.________ company Philip Morris may show the way. Last year it threatened to move its 2,000 head-office employee out 100.________ of the city if the smoking ban became law. Part VI Translation (10 minutes, 20 points) Section A English-Chinese Translation (10 points) Directions: Translate the underlined sentences of the following passage into Chinese on the Answer Sheet. 第 43 页 共 77 页 2003-2010年全国大学生英语竞赛C类初赛、决赛真题及答案汇总集(原创) What is a brand anyway? A brand isn’t just a logo on a shirt or an ice-cream van. It’s much more than that. To be successful a brand has to have rational characteristics—(101)ie, it has to be competitive on price or quality or service, and it has to have a “personality” that charms and seduces. It is a mix of emotional factors,such as “Do I like it?” and “Is it me?” and rational factors such as,“Is it cheaper or better or quicker?”(102)Getting things in balance is tricky,and that’s why so many brands don’t succeed. Just let’s look at all the ways brands can fail. Brands are vulnerable to fashion. Fizzy drinks such as Coke and Pepsi are now being attacked by stimulation drinks such as Red Bull. Fast food brands are threatened by salads and other “lite” foods. McDonald’s has recently been in real trouble. Brands are also vulnerable because they get cocky,arrogant and out of touch. They think they know best and don’t change with the market—like the Gap, Levi’s and Marks & Spencer brands who learnt their lesson the hard way. (103)In fact, even the people who create brands can’t really control them or even predict how people will use them. Not one single mobile phone company anywhere in the world anticipated the growth of texting.The companies involved in the new 3G phone technology are still holding their breath because they don’t know whether it will take off or not. And if it does take off, they won’t know how and in what direction until the market tells them. It’s a huge gamble.Range Rover was the first Sport Utility Vehicle, but Land Rover never completely understood the concept that it had inspired. Range Rover has spent much of its life span trying to catch up with the trend that it serendipitously created. (104)Organic foods are a great success, but not one single major manufacturer or retailer originally promoted them. Little companies started the trend and we consumers just decided we didn’t want our food mucked around with. So at first slowly, hesitantly and ponderously, the retailers and the manufacturers followed. They did what we asked. (105)In other words, brands are nothing like as powerful as they look. The people who manage them often get things wrong, muck things up, look in the wrong direction and generally act just like most organisations directed by human beings—messily. So despite all the huffing and puffing, the reality is that brands are more or less completely in our power. When we like them we buy them, when we don’t, we just buy something else. And what’s more, as customers we’re unpredictable. We can be loyal or fickle, extravagant or stingy, serially or simultaneously. Section B Chinese-English Translation (10 points) 106. 许多遭到洪水侵害的农场主说,他们别无选择只得解雇一些工人。 107. 有时候对一个人来说需要终生的时间才能懂得活着就是为了奉献。 108. 在美国,由于道路畅通,开车上下班很方便,现在许多在城市里工作的人,喜 欢住在乡下。 109. 收音机及电视机使得做广告的人有可能用这种 方法 快递客服问题件处理详细方法山木方法pdf计算方法pdf华与华方法下载八字理论方法下载 吸引千 百万人的注意。 110. 纽约劳动力市场近来显现出从二战以来最长时间的低迷中复苏的迹象。 Part VII Writing (30 minutes, 30 points) Writing Task I (10 points) Directions: Yesterday you lost your student identification card. You need it to get discounts on public transport and for the cinema, and to use the college library. Even more importantly, you need it as proof of identity to withdraw money from the bank. Write to the Director of Student Services explaining the situation and requesting a new card as soon as possible. You should write about 120 words. You do NOT need to write your own address. Write your letter on the Answer Sheet. Begin your letter as follows: 第 44 页 共 77 页 2003-2010年全国大学生英语竞赛C类初赛、决赛真题及答案汇总集(原创) Dear ________, Writing Task II (20 points) Directions: The following is an advertisement for English teachers.Suppose you are a university graduate,and your future plan is to be an English teacher. After reading this advertisement, you decide to write a statement of interest to GEOS. You should write at least 150 words. Write your statement on the Answer Sheet. 2004年全国大学生英语竞赛初赛 听力录音 原文 少年中国说原文俱舍论原文大医精诚原文注音大学原文和译文对照归藏易原文 及参考答案 Part I Listening Comprehension (30 minutes, 30 points) Section A Dialogues (10 points) Directions: In this section, you will hear 10 short dialogues. At the end of each dialogue, a question will be asked about what was said. Both the dialogue and the question will be read only once. After each question,there will be a pause. During the pause, you must read the four choices marked A, B, C and D, and decide which is the best answer. Then mark the corresponding letter on the Answer Sheet with a single line through the centre. 1. W: Hi, I’d like to send this package by express mail to San Francisco and I would like to buy a sheet of stamps, please. M: Here are your stamps, and just put the package on the scale. Q: Where did the conversation take place? (D) 2. M: I’m going out to lunch. Do you need anything while I’m out? W: Yes, if you pass a convenience store, get me some chocolate—a Snickers bar, please. Q: What do you learn from this conversation? (B) 3. W: If we go by car, how do we cross the river? M: There’s a ferry that will take your car. There’s even one for trains. Q: How will they cross the river? (D) 4. W: I heard that the mayor is closing the cheese factory. M: Yes, but it is only temporary. W: Oh, I’m surprised. I thought it was going to shut down for good. Q: Why was the woman surprised? (C) 5. M: I spilled tomato juice on my new white shirt. Do you think it will come out? W: That’s too bad. Leave it there and I’ll see what I can do. Q: What is the man’s problem?(B) 6. W: I’m going to lunch with my bowling instructor. M: What about the committee meeting? W: Don’t worry. I’ll be back at the office before then. Q: Where is the woman probably going now? (C) 7. M: How long have you had this problem with your shoulder? W: It started last week after my skiing accident. M: Let’s try some tests to determine the nature of the injury. Q: What is the man going to do? (B) 8. W: Are you having a good time? M: Sure. Thanks again for inviting me. W: No problem. I just wish more people could have come. 第 45 页 共 77 页 2003-2010年全国大学生英语竞赛C类初赛、决赛真题及答案汇总集(原创) Q: How does the woman feel? (C) 9. M: We finally made it, Mary! W: I can’t believe graduation is tonight. M: Can you come to my graduation party? W: Sure, after I finish the family celebration. M: I want to be sure we get pictures of us together. W: In our caps and gowns! Q: When will the woman go to the man’s graduation party? (A) 10. M: Hi, did you pass your geography exam? W: Yeah, I did quite well in fact, I got 76%. M: Oh,well done! So they gave you a per cent? I thought they gave grades. W: Yeah, they gave both. Mine was an “A”. So how about you? M: Well, we don’t have exams.We have continuous assessment, so you just have to do coursework, and you get a mark for each essay. Q: How does the school evaluate the man’s progress in geography? (A) Section B News Items (10 points) Directions: In this section, you will hear 10 short pieces of news from BBC or VOA. After each news item and question,there will be a pause. During the pause, you must read the three choices marked A, B and C, and decide which is the best answer. Then mark the corresponding letter on the Answer Sheet with a single line through the centre. 11. Tens of thousands of health workers will go house to house over the next three days in an effort to immunise 63 million children under the age of five in sub-Saharan Africa. The campaign is the start of monthly national immunisation days during the low season for polio. It’s hoped that vaccinating children now—when the virus is at its weakest—will be the best way of stopping transmission. Question:How old are the children to be immunised?(B) 12. Amid pomp and ceremony, China launched the 2008 Olympics. Together with a Chinese counterpart, the president of the International Olympic Committee, Jacques Rogge, used a giant golden key to symbolically open what he called the most important market in the world. In his speech, he emphasised the power of the Olympic brand in China’s emerging market. Question:What does the giant golden key symbolize?(C) 13. Microsoft tries to keep the code for its Windows operating system a closely guarded secret. It’s the equivalent of computer DNA and the firm fears if it falls into the wrong hands it could be used to infiltrate millions of computers worldwide. More than 90 percent of the world’s PCs run Windows. Question:What action does Microsoft intend to take?(A) 14. Before he set off in November, there were fears that Francis Joyon would be unable to control his huge boat, named IDEC. With its three hulls slicing through the water and a massive rotating mast that reached 30 metres into the sky, the boat was built in 1986 for a crew of ten. It was feared that such a boat would be too powerful for one man in the rough seas of the Southern Ocean. Question:How many people can the boat carry?(B) 15. Over timescales of thousands of years, the Earth goes through a natural cycle of warmer and colder periods, driven by changes in heat coming from the Sun. Professor William Ruddiman from the University of Virginia has now calculated that if the Earth had followed its natural cycle over 第 46 页 共 77 页 2003-2010年全国大学生英语竞赛C类初赛、决赛真题及答案汇总集(原创) the last ten thousand years, it ought to have got steadily colder. It hasn’t,because, he believes, human activities have been keeping the temperature steady. Question:Has the Earth got steadily colder over the last ten thousand years?(A) 16. Inequality of health care is still paramount, says the WHO’s latest report. Industrialised countries account for less than 20 percent of the world’s population but take 90 percent of health spending. In Japan more than 500 dollars is spent on drugs per person per year. This compares to just three dollars in Sierra Leone. Only slightly more is spent in many sub-Saharan countries. Question: How much do many sub-Saharan countries spend on drugs per person per year?(B) 17. The Iraqi dinar has risen a third or so in value against the dollar since the new banknotes began to circulate. One factor has been the gradual pick up of the Iraqi economy after the devastation of the war. There are simply more transactions taking place, which has supported the value of the currency. And it seems Iraqis trust the new dinar banknotes more than they did the old ones, which featured pictures of Saddam Hussein. Question:Why did the Iraqi new dinar rise in value?(C) 18. The list of countries known to have the relatively new and deadly strain of bird flu is rapidly growing. The focus now is on Indonesia where tests will soon confirm whether or not the bird flu which killed several million chickens there is the often fatal H5N1, already confirmed in 5 other countries in the region. Reports of an outbreak in Laos are also being investigated. Question:What is the number of countries mentioned in this news report?(C) 19. An unhealthy diet together with little exercise and smoking are the key preventable risks of non-communicable diseases and it’s estimated that low fruit and vegetable intake alone causes more than two and a half million deaths each year. Question:What causes more than two and a half million deaths each year?(A) 20. Around Europe interest rates are at their lowest levels in half a century. But businesses are pressing for even cheaper borrowing costs amid signs of continued economic weakness. A big drop in German manufacturing announced earlier this week is cited as evidence that Europe’s most important economy may even be sliding into recession. And the rise of the euro to a four-year high against the dollar in currency dealing is a major worry for many European exporters. Question:What is the key problem for European exporters?(A) Section C Passages (10 points) Directions:In this section, you will hear 2 passages. At the end of each passage, you will hear 5 questions. After you hear a question, you must choose the best answer from the four choices marked A, B, C and D. Then mark the corresponding letter on the Answer Sheet with a single line through the centre. Passage One The world of music will never be the same since the formation of a band in Liverpool, England in 1956. The Beatles were formed by George Harrison, Ringo Starr, Paul Mc-Cartney, and John Lennon. Their first hit song Love Me Do was recorded in 1962. The Beatles quickly became the world’s best-known pop music group and many people today still regard them as the finest band in the history of pop music. Lennon and McCartney were the authors of most of the songs the group recorded. Harrison also wrote songs, often using ideas from Indian music. The drummer of the group was the famous Ringo Starr and he occasionally sang. For six years the Beatles had hit after hit song. Twenty-eight 第 47 页 共 77 页 2003-2010年全国大学生英语竞赛C类初赛、决赛真题及答案汇总集(原创) of their songs were on the Top Twenty record charts and seventeen of these songs reached number one on the charts. The group also had a successful movie career. The comedies A Hard Day’s Night and Yellow Submarine became very successful movies. People imitated their long hairstyles, clothing, and humor. Almost all later pop bands learned from the Beatles. Beatlemania is the word used to describe how strong and loyal the fans were. Questions 21 to 25 are based on the passage you have just heard: 21. What kind of music did the Beatles play?(D) 22. What did many people copy from the Beatles?(D) 23. Where were the members of the Beatles group from?(B) 24. Which of the following is NOT true?(C) 25. How many of the Beatles’ songs reached number one on the record charts?(A) Passage Two Have you ever wondered where these cute little teddy bears came from? They were named for President Theodore Roosevelt in 1902. President Roosevelt was on a hunting trip in Mississippi when members of the hunting party caught a black bear and tied him to a tree. President Roosevelt was called to the area to shoot the bear, which he refused to do and said it was unsportsmanlike and showed poor manners. The Washington Post newspaper ran a cartoon showing the President refusing to shoot the bear and people all over America saw the cartoon. Morris Michtom, a shopkeeper in Brooklyn, New York, placed two toy bears in the window of his shop. Mr. Michtom requested permission from the President to call them “Teddy Bears” as Teddy is the nickname for Theodore Roosevelt. The sweet little bears with shiny button eyes were a delight with children everywhere. The Teddy Bears were made by Mr. Michtom’s wife. Mr. Michtom formed a new business called the Ideal Novelty and Toy Corporation. Today, Teddy Bears are treasured toys of children all over the world. They are also collected by people and many are displayed in museums. Teddy Bears are sold by many companies and you can find them in almost any toy store, dressed in costumes or with a ribbon around the neck. Questions 26 to 30 are based on the passage you have just heard: 26. Why did President Roosevelt refuse to shoot the bear?(C) 27. Why did Mr. Michtom ask for the President’s permission to call the toy bears “Teddy Bears”?(A) 28. Which of the following is NOT true?(D) 29. How many Teddy Bears were made by Mrs. Mitchtom and placed in the window of their shop?(C) 30. What did Mr. Mitchtom do after he sold the Teddy Bears in 1902?(D) Part II Vocabulary and Structure (10 minutes, 20 points) Section A Multiple Choice (10 points) 31. A 32. D 33. A 34. C 35. B 36. C 37. B 38. D 39. B 40. C Section B Cloze-Test (10 points) 41. B 42. A 43. A 44. B 45. A 46. B 47. C 48. D 49. D 50. A Part III Word Guessing and IQ Test (5 minutes, 10 points) Section A Word Guessing (5 points) 51. B 52. B 53. D 54. C 55. B 第 48 页 共 77 页 2003-2010年全国大学生英语竞赛C类初赛、决赛真题及答案汇总集(原创) Section B IQ Test (5 points) 56. C 57. A 58. B 59. A 60. A Part IV Reading Comprehension (25 minutes,30 points) 61. trays 62. To preserve their colours. (or: To prevent darkening.) 63. In hot-air chambers. 64. dried separately and then mixed 65. climbers, explorers, soldiers 66. Because it takes so little time to cook them. 67. The travails of comics connoisseur Harvey Pekar. 68. original screenplay 69. Los Angeles, New York 70. Encouraged and excited. 71. Bend It Like Beckham, Dirty Pretty Things, In America, The Station Agent. ( Any three of them.) 72. 15. 73. modern advances in surgery 74. the stomach or one lung 75. 20% 76. The body’s tendency to reject alien tissues. 77. No, it has yet to become a reality. 78. your illness may be curable 79. tripled 80. Leeds 81. Manchester 82. Married women, those unmarried with partners 83. “Sindies”, women in their 40s 84. The sales have reached a new high, with regional variations. 85. Dress, way of speaking, area of residence education and manners. (Any three of them.) 86. Rulers, administrators, freemen and slaves. 87. politically 88. recurrent 89. resident foreigners 90. The rise of the burghers. Part V Error Correction (5 minutes,10 points) 91. non-smoke?non-smoking 92. also ? smoked?be 93. smoke?smokeless / non-smoking 94. banned?banning 95. to?from 96. down?up 97. has?has 98. ? 99. economical?economic 第 49 页 共 77 页 2003-2010年全国大学生英语竞赛C类初赛、决赛真题及答案汇总集(原创) 100. employee?employees Part VI Translation (10 minutes, 20 points) Section A English-Chinese Translation(10 points) 101. 即它必须在价格或质量或服务方面具有竞争力,并且还应具有能够吸引人 们购买的“个性特点”。 102. 保持诸方面因素的平衡是很需要技巧的,这也正是如此多的品牌不成功的 原因所在。 103. 事实上,即使是设计品牌的人也无法左右品牌,他们甚至预测不 出人们将如何利用它们。 104. 有机食品是很成功的, 但在最初却没有一个较大 规模的生产商或零售商(愿意)推销它们。 105. 换句话说,品牌本身远没有他们看起来那样有影响力。 Section B Chinese-English Translation (10 points) 106. Many flood-ravaged farmers say they will have no choice but to lay off some employees. 107. Sometimes it takes a lifetime for one to realize that to live is to give. 108. Many people in America who work in the cities now enjoy living in the country because of good roads and the ease of commuting by automobile. 109. Radio and television have made it possible for advertisers to capture the attention of millions of people in this way. 110.The labor market of New York has recently shown signs of emerging from its longest slump since World War II. Part VII Writing (30 minutes, 30 points) Writing Task I (10 points) One possible version: Dear Director, I have to report the loss of my student identification card yesterday, Tuesday, September 3rd. I am a student in Business Studies. I believe I lost my card at the swimming pool when I dropped my backpack and many items fell to the ground. I need my card to get money from the bank. The bank will only accept my student card as ID. I asked the bank if they would accept my passport, but they said that my student ID card was used to open the account, and that I should bring it with me when I want to withdraw money. I also want to go to the cinema, but I will not get a concession without my student card. Could you please issue me a new card as soon as possible? I will come to your office this afternoon to see if I can get one immediately. Thank you very much. Yours sincerely, ### Writing Task II (20 points) One possible version: Dear Sir or Madam: I am currently a student at # # University. After reading your message in # # newspaper, I am very interested to apply for a teaching position with GEOS. English has become a world language, spoken and understood everywhere in the world. Therefore, to be an English teacher in foreign countries is to experience different cultures. Some of my friends, who are from the UK and are teaching English here, regard teaching English not only as a career but also a chance to learn about people and culture. They set good examples for me, and 第 50 页 共 77 页 2003-2010年全国大学生英语竞赛C类初赛、决赛真题及答案汇总集(原创) they have encouraged me to apply for a chance to join you. English is one of my favorite subjects. To be an English teacher always has been my dream. After four years of study in this university, I have passed the College English Test Band 6. Most of the courses I have taken are specially focused on teaching methods and skill training. If you are looking for an energetic, outgoing and motivated university graduate, I am sure you have found one. Sincerely yours, ### 2008年全国大学生英语竞赛样题(C级) Part IListening Comprehension (25 minutes, 30 marks) Section A(5 marks) Directions: In this section, you will hear 5 short conversations. At the end of each conversation, a question will be asked about what was said. Both the conversation and the question will be read only once. After each question, there will be a pause. During the pause, you must read the three choices marked A, B and C, and decide which is the best answer. Then mark the corresponding letter on the Answer Sheet with a single line through the centre. 1. A. The man is not suitable for the position. B. The job has been given to someone else. C. She hadn’t received the man’s application. 2. A. He is going to see his section chief. B. He is going to have a job interview. C. He is going to see his girlfriend. 3. A. Ask to see the man’s ID card. B. Get the briefcase for the man. C. Show the man her documents. 4. A. The dorm room is too crowded. B. There is no kitchen in the building. C. No one looks after the dorm building. 5. A. She was always in good shape. B. She stopped exercising one year ago. C. She lost a lot of weight in one year. Section B (10 marks) Directions: In this section, you will hear two long conversations. Each conversation will be read only once. At the end of each conversation, there will be a one-minute pause. During the pause, you must read the five questions, each with three choices marked A, B and C, and decide which is the best answer. Then mark the corresponding letter on the Answer Sheet with a single line through the centre. Conversation One 6. What is soon to open in China? A. The French Movie Festival. B. The French Cultural Year. C. The French Food Festival. 7. How many exhibitions will be held for this activity? A. 200.B. 20.C. 100. 第 51 页 共 77 页 2003-2010年全国大学生英语竞赛C类初赛、决赛真题及答案汇总集(原创) 8. What will be held at the foot of the Great Wall? A. The City Concert. B. The Opening Ceremony. C. The Great Lunch. 9. Which of the following cities is not included in this activity? A. Chongqing. B. Wuhan. C. Shenzhen. 10. What will certainly make great contributions to this activity? A. Internet. B. TV shows. C. Newspapers. Conversation Two 11. What sound more like a native speaker in a casual conversation? A. Examples. B. Verbs. C. Idioms. 12. Which of the following sounds more informal and more natural? A. Get together. B. Meet. C. See. 13. Which of the following means that you cannot interrupt me? A. I’m tied up. B. I have a lot on my plate. C. I’m busy. 14. In American culture, what is considered important in a conversation? A. Using proper language B. Making eye contact. C. Looking at your own feet. 15. In business, how might Americans feel about you if you are looking away? A. You’re feeling ashamed. B. You’re telling the truth. C. You’re telling a lie. Section C (5 marks) Directions: In this section, you will hear 5 short news items. After each item, there will be a pause. During the pause, you must read the question and then the three choices marked A, B and C, and decide which is the best answer. Then mark the corresponding letter on the Answer Sheet with a single line through the centre. 16. How long will it take for new forests to grow back? A. 65 years. B. 20 years. C. 40 years. 17. What did Jimmy Carter plan to focus his efforts on after leaving the White House in 1981? A. A presidential library. B. Camp David. C. Winning a second term. 18. What was regarded as the lifeblood of the country of the Maldives? A. Oil. B. Agriculture. C. Tourism. 19. What is responsible for the death of many people in developing countries? 第 52 页 共 77 页 2003-2010年全国大学生英语竞赛C类初赛、决赛真题及答案汇总集(原创) A. The development of resistance to diseases. B. The difficulty to cure new emerging diseases. C. The inability of the poor to afford medicine. 20. What released an estimated 8.7 million tons of the global warming gas? A. Cars. B. Wildfires. C. Wars. Section D (10 marks) Directions: In this section, you will hear a short passage. There are ten missing words or phrases in it. Fill in the blanks with the exact words you hear on the tape. Remember to write the answers on the Answer Sheet. A researcher says lead in the environment could be a major cause of violence by young people. Doctor Herbert Needleman is a (21) ________ at the University of Pittsburgh School of Medicine in Pennsylvania and he (22) ________ his findings at the yearly meeting of the American Association for the Advancement of Science. Doctor Needleman says the presence of lead in the (23) ________ changes the neurons that control actions and that can cause a person to act in antisocial and (24) ________ ways. In the 1970s, Doctor Needleman found lower scores on (25) ________ even in children who did not have such signs of lead poisoning. After that, lead was (26)________ gasoline and paint in the United States. Yet many homes still have old lead paint. Lead was also used in older (27) ________. In fact, officials just announced stronger testing and reporting requirements as from next year for lead in American drinking water. The newest research shows that even very small amounts of lead in bones can affect brain development. A simple (28)________ can measure lead except that an X-ray process is needed to measure levels in bone. In 2004, such tests were done on 190 young people who were (29) ________ and the findings showed that their average levels were higher than normal. And, in 1998, three hundred children were studied and the test scores showed higher levels of (30) ________ problems in those with increased levels of lead. Yet these levels were still considered safe by the government. Part II Vocabulary and Structure (10 minutes, 15 marks) Directions: There are 15 incomplete sentences in this part. For each blank there are four choices marked A, B, C and D. Choose the one that best completes the sentence. Then mark the corresponding letter on the Answer Sheet with a single line through the centre. 31. So nervous ________ that she didn’t know how to start her speech. A. since she became B. would she become C. that she became D. did she become 32. He ________ another career but, at the time, he just wanted to earn money to study abroad. A. might have chosen B. might choose C. had to choose D. must have chosen 33. The second report was ________ by August 2005, but one year later it was still nowhere in sight. A. submitted B. to have submitted C. to submit D. to have been submitted 第 53 页 共 77 页 2003-2010年全国大学生英语竞赛C类初赛、决赛真题及答案汇总集(原创) 34. In this experiment, the students studied are stopped several times during the listening test and asked to report what they ________ during the pause before answering the questions. A. had just been thinking about B. have just been thinking about C. are just thinking about D. had just thought about 35. I was always taught that it was ________ to interrupt. A. rude B. coarse C. rough D. crude 36. Small boys are ________ questioners. They ask questions all the time. A. original B. peculiar C. imaginative D. persistent 37. We regret to inform you that the materials you ordered are ________. A. out of work B. out of reach C. out of stock D. out of practice 38. The bomb will ________ the moment it is touched. A. go on B. go off C. go out D. go over 39. The car won’t ________; I’ve tried it several times, but it won’t work. A. begin B. launch C. start D. drive 40. Children and old people do not like having their daily ________ upset. A. habit B. routine C. practice D. custom 41. In your first few days at school you’ll be given a test to help the teachers to ________ you to a class at your level. A. locate B. assign C. deliver D. place 42. China only started its nuclear power industry in recent years, and should ________ no time in catching up. A.lose B.delay C. spare D. relieve 43. — You did an excellent job yesterday, Jim! I really enjoyed your presentation. — ________ — Oh yeah, it was fabulous. It seems the English program is a great way to practice English. — Yeah. It is fun and motivating. A. Did you really? B. Oh, thank you. You are so kind. C. Really? What about yours? D. Not at all. My pleasure. 44. — What kind of music do you like? — Well, I like different kinds. — ________ — Er, I especially like punk rock. A. I beg your pardon? B. Are you serious? C. Any in particular? D. Why do you think so? 45. — How did you like the fashion show last night? — ________ — I didn’t see anything wrong with the clothes; they looked pretty nice to me. — Do you really think people can wear that stuff and walk around in streets? A. Impressive. It’s a good way to show off women’s sense of style and wealth. B. It was cool. The clothes are more beautiful than the people wearing them. C. Nothing serious. It’s only a show to attract the eyes of fashion fans. D. It was dumb. I think it’s stupid for women to wear clothes like that. 第 54 页 共 77 页 2003-2010年全国大学生英语竞赛C类初赛、决赛真题及答案汇总集(原创) Part III Reading Comprehension (20 minutes, 40 marks) Section A (4 marks) Directions: There is one passage in this section with 4 questions. For each question, there are four choices marked A, B, C and D. You should decide on the best choice. Then mark the corresponding letter on the Answer Sheet with a single line through the centre. Questions 46-49 are based on the following passage. The ability to “see” oneself in the future is a remarkable human trait - some would say unique - that is not well understood. That’s despite the fact that we probably spend as much time thinking about the future as we do thinking about the present. Now new research from Washington University in St. Louis suggests that it’s precisely because we can remember the past that we can visualize the future. “Our findings provide convincing support for the idea that memory and future thought are highly interrelated and help explain why future thought may be impossible without memories,” says doctoral candidate Karl Szpunar. The findings are consistent with other research showing that persons with little memory of the past, such as young children or individuals suffering from loss of memory, are less able to see themselves in the future. The researchers base their conclusions on brain scans of 21 college students who were cued to think about something in their past, and anticipate the same event in the future, like a birthday or getting lost. The experiment was carried out as each student lay on their stomach in a magnetic resonance imaging machine, a dreadful but very useful piece of equipment that can show which areas of the brain are stimulated during specific thought processes. The students were also asked to picture former President Bill Clinton in a past and future setting. Clinton was chosen because he was easily recognized and familiar to all the students. The researchers found a “surprisingly complete overlap” among regions of the brain used for remembering the student’s past and those used for picturing the future. And every region involved in remembering was also used in anticipating the future. In short, the researchers isolated the area of the brain that “lit up” when the students thought about an event in their own past. And more importantly, that same area lit up again when they thought about a similar event in their future. In fact, the researchers report that the brain activity was so similar in both cases that it was “indistinguishable.” The findings were reinforced when students imagined Bill Clinton. Since none of them knew him personally, their memories were not autobiographical. And the brain scans showed “significantly less” correlation between memories of having seen pictures of Clinton in the White House and projecting him into the future. So this “time machine,” as the researchers describe it, allows us to use the past to see ourselves in the future, and both our memories and our anticipation are interdependent. 46. A remarkable human trait that is not well understood is the ability ________. A. to think about the past B. to see the future C. to remember the past D. to control the present 47. The findings support that ________. A. future goals will greatly influence a person’s present performance B. a person’s present performance is determined by his / her past knowledge C. future thought depends to a great degree on the memory of the past D. present thought is impossible without the ability to imagine the future 第 55 页 共 77 页 2003-2010年全国大学生英语竞赛C类初赛、决赛真题及答案汇总集(原创) 48. The conclusion of the experiment on students was that ________. A. the students could picture themselves better than Bill Clinton in a past and future setting B. the students could imagine themselves as well as Bill Clinton in a past and future setting C. the students could anticipate Bill Clinton better than themselves in a past and future setting D. the students could only picture themselves in a past and future setting but not Bill Clinton 49. This “time machine” in the last paragraph most probably refers to ________. A. clock B. brain scanning C. magnetic resonance imaging D. memory Section B (14 marks) Directions: There is one passage in this section with 10 questions. Go over the passage quickly and answer the questions on the Answer Sheet. For questions 50-55, mark Y (for YES)if the statement agrees with the information given in the passage; N (for NO)if the statement contradicts the information given in the passage; NG (for NOT GIVEN)if the information is not given in the passage. For questions 56-59, complete the sentences with the information given in the passage. Visiting the White House White House Tours Public tours of the White House are available for groups of 10 or more people. Requests must be submitted through one’s Member of Congress and are accepted up to six months in advance. These self-guided tours are available from 7:30 a.m. to 12:30 p.m. Tuesday through Saturday , and are scheduled on a first come, first served basis approximately one month in advance of the requested date. We encourage you to submit your request as early as possible since a limited number of tours are available. All White House tours are free of charge. For the most current tour information, please call the 24-hour line at 202-456-7041. Please note that White House tours may be subject to last minute cancellation. White House Visitor Center All tours are significantly enhanced if visitors stop by the White House Visitor Center located at the southeast corner of 15th and E Streets, before or after their tour. The Center is open seven days a week from 7:30 a.m. until 4:00 p.m. and features many aspects of the White House, including its architecture, furnishings, first families, social events, and relations with the press and world leaders, as well as a thirty-minute video. Allow between 20 minutes to one hour to explore the exhibits. The White House Historical Association also sponsors a sales area. Please note that restrooms are available, but food service is not. Mobility-Impaired / Using a Wheelchair Guests requiring the loan of a wheelchair should notify the officer at the Visitors Entrance Building upon arrival. Wheelchairs loans are offered on a first-come, first-served basis. Reservations are not possible. Visitors in wheelchairs, or with other mobility disabilities, on the Congressional guided or self-guided tours, between 8:00 a.m. and 12 noon, use the same Visitor entrance and, with up to four members of their party, are admitted without waiting in line and without tickets. Visitors in wheelchairs are escorted by ramp from the entrance level to the ground floor, and by elevator from the ground to the state floor. Guests generally wait in line with their family or 第 56 页 共 77 页 2003-2010年全国大学生英语竞赛C类初赛、决赛真题及答案汇总集(原创) group. Hearing-Impaired Tours for hearing-impaired groups may be arranged in advance by writing to the Visitors Office, White House, Washington, DC 20502. Tours are usually scheduled at 9:30 a.m., between the Congressional and public tour times. Participants enter at the East Appointment gate. A U.S. Secret Service / Uniformed Division Tour Officer conducts the tour in sign language. Signed tours are available to groups of 8 to 20. Groups are also encouraged to bring their own interpreters. Signing interpretation is also available for individual visitors with advance notice. A Congressional office first issues guided tour tickets to a guest who is hearing-impaired and then contacts the Visitors Office at least 2 weeks in advance to request interpreter service. The Visitors Office TDD (telephone device for the deaf) is 202-456-2121. Messages may be left outside normal business hours. Visually-Impaired Tours for visually-impaired groups may be arranged in advance by writing to the Visitors Office, White House, Washington, DC 20502. The tours are usually scheduled at 9:30 a.m., between the Congressional and public tour times. Participants enter at the East Appointment gate. A U.S. Secret Service / Uniformed Division Tour Officer permits visitors to touch specific objects in the House. Touch tours are currently available only to groups of 8 to 20, not to individual visitors. Guide animals are permitted in the White House. General Tour Information All White House tours are free. Changes in tour schedules are occasionally made because of official events. Notice may not be given until that morning. The Visitors Office 24-hour Information Line recording at 202-456-7041 provides the most up-to-date information. The TDD is 202-456-2121. Visitors should confirm tour schedules by calling the information line the night before and the morning that they plan to visit. It is occasionally necessary to close individual rooms on the tour; however, notice about closed rooms is not possible. Prohibited Items Prohibited items include, but are not limited to, the following: handbags, book bags, backpacks, purses, food and beverages of any kind, strollers, cameras, video recorders or any type of recording device, tobacco products, personal grooming items (make-up, hair brush or comb, lip or hand lotions, etc.), any pointed objects (pens, knitting needles, etc.), aerosol containers, guns, ammunition, fireworks, electric stun guns, mace, martial arts weapons / devices, or knives of any size. The U.S. Secret Service reserves the right to prohibit any other personal items. Umbrellas, wallets, cell phones and car keys are permitted. Please note that no storage facilities are available on or around the complex. Individuals who arrive with prohibited items will not be permitted to enter the White House. Parking The closest Metrorail stations to the White House are Federal Triangle (blue and orange lines), Metro Center (blue, orange, and red lines) and McPherson Square (blue and orange lines). On-street parking is not available near the White House, and use of public transportation is strongly encouraged. Restrooms / Public Telephones The nearest restrooms and public telephones to the White House are in the Ellipse Visitor Pavilion (the park area south of the White House) and in the White House Visitor Center. 第 57 页 共 77 页 2003-2010年全国大学生英语竞赛C类初赛、决赛真题及答案汇总集(原创) Restrooms or public telephones are not available at the White House. 50. Both Congressional guided and self-guided tours need to be scheduled in advance. 51. All White House tours are free of charge except on federal holidays. 52. The White House Visitor Center provides free drinks but not food service. 53. Wheelchair reservation service is provided by the officer at the Visitors Entrance Building. 54. Hearing-impaired visitors can request signing interpretation service from the Visitors Office. 55. Touch tours are currently only offered to visually-impaired groups of 8 to 20. 56. Sometimes official events make it necessary to close ________________ without notice. 57. The personal items permitted to be carried into the White House are ________________. 58. The transportation visitors are encouraged to use is ________________. 59. Inside the White House, visitors cannot find or use restrooms or ________________. Section C (10 marks) Directions: In this section, there is one passage followed by 5 questions. Read the passage carefully, then answer the questions in as few words as possible (not more than 10 words). Remember to write the answers on the Answer Sheet. Questions 60-64 are based on the following passage. If you were on a distant planet, and if you had instruments that could tell you the composition of Earth’s atmosphere, how would you know there was life on this planet? Water in the atmosphere would suggest there could be water on the surface, and as we all know water is considered crucial to life. But water would only suggest that life is possible. It wouldn’t prove it’s there. Carbon? That basic component of “life as we know it?” Not necessarily. A diamond is pure carbon, and it may be pretty, but it isn’t alive. What really sets Earth apart is nitrogen, which makes up 80 percent of the planet’s atmosphere. And it’s there only because there is abundant life on Earth, say scientists at the University of Southern California. The report grew out of a class discussion two years ago in a course taught by Capone and Kenneth Nealson, professor of earth sciences. Students were asked to come up with different ideas about searching for life on other planets. What is a distinct “signature,” as Capone puts it, that would show there is life on another planet? That’s a question that has been kicked around in many quarters in recent decades, especially since all efforts to find some form of life, no matter whether on Mars or in the distant reaches of space, have failed. At least so far. The current effort to search for some evidence of life on Mars focuses primarily on the search for water, because it has long been believed that water, or at least some fluid, is necessary for the chemical processes that lead life to take place. But that’s probably the wrong approach, the USC group argues. “It’s hard to imagine life without water, but it’s easy to imagine water without life,” says Nealson, who was on the Mars team before moving to USC. But nitrogen would be a much clearer signature of life. Only about 2 percent to 3 percent of the Martian atmosphere is nitrogen. That’s just a trace, and it probably means there is no life on Mars today, and if there was in the past, it probably ended many, many years ago. 第 58 页 共 77 页 2003-2010年全国大学生英语竞赛C类初赛、决赛真题及答案汇总集(原创) But, the USC team adds quickly, that doesn’t mean there’s no life anywhere else in the universe. They don’t know where, of course, but they may have found a way to narrow down the search. Look first for nitrogen, then look for biological activity that should be there. So if life exists elsewhere, and is similar to life as we know it, there should be nitrogen, and that’s what we should be looking for first, the researchers say. If they don’t find nitrogen on Mars, Capone says, “that will probably bring us to the conclusion that there likely never was life on Mars.” But how about elsewhere? Could this technique be used to search for life in other solar systems? Maybe. It might be possible to detect a nitrogen-rich atmosphere around a planet orbiting another star, but not yet. Current instruments aren’t that sensitive. If they ever are, the search for life might be narrowed down to the most promising prospects, chiefly because of the presence of nitrogen. And won’t that be fun! Questions: 60. What can suggest life is possible but cannot be proved according to the author? 61. What is a clear “signature” of life on another planet according to Capone? 62. What is considered as a wrong way to search for evidence of life on Mars? 63. What can probably prove there is no life on Mars today based on the new theory? 64. Why is it impossible to use the new technique to search for life in other solar systems now? Section D (12 marks) Directions: In this section, there is one passage followed by a summary. Read the passage carefully and complete the summary below by choosing a maximum of three words from the passage to fill in the spaces 65-70. Remember to write the answers on the Answer Sheet. Questions 65-70 are based on the following passage. In August 2008, athletes from the United States and around the world will compete in the Beijing Olympics. But did you know that in September of next year, disabled athletes will compete in the Paralympic Games in Beijing? The Olympics and the Paralympics are separate movements. But they have always been held in the same year, and since 1988, they have also been held in the same city. The International Olympic Committee and the International Paralympic Committee signed an agreement in 2001 to secure this connection. The next winter games will take place in Vancouver, Canada, in 2010. The Paralympic Games grew out of a sports competition held in 1948 in England and a doctor named Ludwig Guttmann organized it for men who suffered spinal cord injuries in World War II. Four years later, it became an international event as competitors from the Netherlands took part. Then, in 1960, the first Paralympics were held in Rome. 400 athletes from 23 countries competed. By 2004, the Paralympic Games in Athens had almost 4000 athletes from 136 countries, who may have physical or mental limitations and may be blind or in wheelchairs. Yet sometimes they perform better than athletes without disabilities. In 1968, Eunice Kennedy Shriver, the sister of former President John F. Kennedy, started the Special Olympics, which are just for children and adults with mental limitations and whose programs currently serve more than two million people in 160 countries. In November 2006, in Mumbai, India, teams competed in the First Special Olympics International Cricket Cup. In addition to India, there were men’s teams from Afghanistan, Australia, Bangladesh, Nepal, 第 59 页 共 77 页 2003-2010年全国大学生英语竞赛C类初赛、决赛真题及答案汇总集(原创) Pakistan, Sri Lanka and the West Indies. There were also women’s cricket teams from India and Pakistan. There are many organizations in the United States that help people with disabilities play sports. Wheelchair tennis is a popular sport. So is basketball. In fact, there are more than one hundred professional teams playing wheelchair basketball thanks to the special wheelchairs for athletes that are lightweight and designed for quick moves. For people who want to go really fast in their chairs, there is a Power Wheelchair Racing Association. In the state of Utah there is a place called the National Ability Center, which teaches all kinds of sports to people with all kinds of physical and mental disabilities and even gives friends and family members a chance to try a sport as if they were disabled. A reporter from the Washington Post wanted to know what it would be like for a blind person to use a climbing wall. So, protected by a safety line, the newspaper reporter closed his eyes and started to feel for places to put his hands and feet. Trainers on the ground urged him on: “Take your time. You can do it.” Finally he reached the top. At the National Ability Center people can learn to ride horses and mountain bikes. They can try winter mountain sports, and learn scuba diving and other water activities. The center also prepares athletes for the Paralympics. These days, the first place many people go when they want to travel is the Internet, where they can get information about hotels, transportation and services like tour companies. The Internet can also help travelers find special services for the disabled. For example, there are groups that help young people with disabilities travel to different countries. Susan Sygall, who uses a wheelchair herself, leads an organization called Mobility International USA, and has traveled to more than twenty-five countries to talk about the rights of people with disabilities. She says people with disabilities are all members of a global family and working together across borders is the most powerful way of making changes. Summary: The Olympics and the Paralympics are (65) ________ but they have always been held in the same year and also in the same city since 1988 when the International Olympic Committee and the International Paralympic Committee signed an agreement in 2001 to (66) ________. The Paralympic Games grew out of a sports competition organized by a doctor named (67) ________ in 1948 in England for men injured in World War II. In 1952, it became an (68)________ and in 1960, the first Paralympics were held in Rome for people who may have physical or mental limitations or may be blind or in wheelchairs. The (69) ________ was started in 1968 in the United States by Eunice Kennedy Shriver just for children and adults with mental limitations and to help people with disabilities play sports and enjoy other activities, many (70) ________ are founded, such as the Power Wheelchair Racing Association, the National Ability Center and Mobility International USA. Part IV Cloze (15 minutes, 15 marks) Section A: There are 5 blanks in the passage. Use the word given on the right side to form a word that fits in each blank. Remember to write the answers on the Answer Sheet. Maria Callas was one of the best-known opera singers in the world, who became famous internationally for her beautiful voice and intense (71)________ during the 1950s, and the recordings of her singing the well-known operas remain very popular today. Maria Callas was born in New York City in 1923 and her real name was Maria Kalogeropoulous. 第 60 页 共 77 页 2003-2010年全国大学生英语竞赛C类初赛、决赛真题及答案汇总集(原创) Her parents were Greek and when she was fourteen, she and her mother returned to Greece, where Maria studied singing at the national conservatory in Athens and the well-known opera (72)________ Elvira de Hidalgo chose Maria as her student. In 1941, when she was 17, Maria Callas was paid to sing in a major opera for the first time. She sang the (73)________ role in several operas in Athens during the next three years. In 1943, Callas was invited to perform in Italy, which was the real beginning of her profession as an opera singer. She performed major parts in several of the most (74)________ operas. In 1949, she married an Italian (75)________, Giovanni Battista Meneghini, who was twenty years older and became her adviser and manager. Person sing lead fame industry Section B: There are 10 blanks in the passage. For each blank, some letters of the word has been given (not exceeding 3 letters). Read the passage below and think of the word which best fits each blank. Use only one word in each blank. Remember to write the answers on the Answer Sheet. Autism is a general (76)________ for a group of brain disorders that limit the development of social and communication skills, t m which (77)________ professionals call autism spectrum disorders. me l Experts say autism is permanent and cannot be cured. But there are ways to treat it that they say can (78) ________ the re e severity, and the academy says the earlier treatment begins, the(79)________ the results. b r The medical group released two reports Monday with detailed information to help doctors (80)________autism. Chris Johnson id y at the University of Texas Health Science Center in San Antonio was one of the authors, who says doctors should look for signs of autism when they (81)________ babies at eighteen months and ex e twenty-four months. Doctors traditionally (82)________ the possibility of co r autism only if a child shows delayed (83)________ or unusually sp h repetitive behaviors. These may be clear signs of it, but they usually do not appear until a child is two or three years old. Parents could answer a list of written questions about their baby, and then the doctor could (84) ________ tests as pe m simple as observing the baby’s ability to follow a moving object with its eyes. Experts say failing to watch a moving object may be a sign of autism. Doctors and parents can also look for behaviors that are normal in babies under one year of age. For example, does the baby appear to (85)________ to a parent’s voice? Does the baby make eye contact? Does the baby wave or point at things? Part V Translation (15 minutes, 15 marks) Section A (8 marks) Directions: Translate the underlined sentences of the following passage into Chinese. Remember to write the answers on the Answer Sheet. When you talk about China and India’s seemingly unstoppable growth, nobody’s surprised. 第 61 页 共 77 页 re d 2003-2010年全国大学生英语竞赛C类初赛、决赛真题及答案汇总集(原创) After all, the increasing economic strength of the two countries has dominated the news for the past few years. But both China and India face some substantial demographic hurdles to continued expansion. Despite the billion-plus citizens of each country, both may simply lack sufficient qualified workers. (86) China is a rapidly aging society whose current challenges of unemployment and overpopulation will, within a decade, shift to different problems: labor shortages and an elderly population with too few children. In fact, China may be the first country to go gray before it reaches developed status. The World Bank estimates that by 2020 the mainland will face a lack of even unskilled labor due to aging. China is already facing a shortage of skilled labor. Construction sites lack welders, skilled machine operators, and plumbers. And a recent report said the country is short some 750,000 managers. Despite a 95% literacy rate among all but the oldest citizens, there are not enough well-educated Chinese. (87) This is in part due to a lack of schools that combine basic theory with practical skills and a focus on passing the elite university exams. So despite rising salaries, many of those entering China’s workforce cannot learn the skills they need. India seems to have an age advantage, with half its population under 25. In the long run, this gives them the upper hand. But at present, India shares China’s problem of an insufficiently educated workforce. Citigroup reports that India’s talent pool isn’t deep enough to meet demand in industries including textiles, aviation, telecom, retail, and engineering. By some other measures, both countries look pretty good. China graduates about 1.7 million students from 1,500 colleges and universities annually, of whom 350,000 are engineers. India produces about 3 million college and university graduates, including 440,000 engineers. Those numbers, though, don’t tell the whole story.(88) Only 10% to 25% of these graduates are employable by multinationals, and this is only partially because of language challenges, which every human resources executive I’ve met who works in China or India agrees with. The source of the problem is cultural. Young people in these countries are highly motivated to study and learn, but only in theory. There is no tradition of practical application. So engineers don’t go into operations, factories, or mines, and don’t really know the machines and conditions for which they are designing processes. (89) Chinese and Indian students who have just earned BAs want to pursue MBAs right away, and neither they nor their parents understand the value of practical experience. The degree is seen as a trophy, a traditional ticket to higher status. Section B (7 marks) Directions: Translate the following sentences into English, using the words given in the brackets. Remember to write your answers on the Answer Sheet. 90. 如果你告诉他们真相,你就可以获得这次机会而不被他们误解。(without) 91. 不管多难,我也要及时完成文献 翻译 阿房宫赋翻译下载德汉翻译pdf阿房宫赋翻译下载阿房宫赋翻译下载翻译理论.doc 。(no matter) 92. 中国在提高农村居民生活水平方面已经取得了很大成绩,这一事实谁也无 法否认。(deny) Part VI IQ Test (5 minutes, 5 marks) Directions: There are 5 IQ Test questions in this part. Write your answer on the Answer Sheet. 93. Live is to Evil just as 5423 is to ________. 94. One half of one quarter of one tenth of 800 is ________. 95. The number that should come next in the series 64, 16, 4, 1, 1/4 is ________. 第 62 页 共 77 页 2003-2010年全国大学生英语竞赛C类初赛、决赛真题及答案汇总集(原创) 96. Ann is taller than Jill, and Kelly is taller than Ann. Then we can draw the conclusion that Jill is definitely ________ Kelly. 97. One boy and a girl caught 40 frogs. Jack caught four times as many as Jen did. The number of frogs caught by Jen was ________. Part VII Writing (30 minutes, 30 marks) Task I (10 marks) Directions: You are required to write a Notice in about 100 words to inform the students and English teachers of a lecture on American Liberal Education by Prof. Gilbert in the English Department Hall. Please write it on the Answer Sheet. Task II (20 marks) Directions: For this part, you are required to write a composition on The Anti-Addiction System. You should write at least 120 words and please write it on the Answer Sheet. Now in China the anti-addiction system and a “real name checking” system have been adopted in Cyber cafes to prevent players under 18 from becoming addicted to online games. What do students think about this system? What is your opinion about this? 参考答案 Part I Listening Comprehension Section A 1. B 2. A 3. B 4. A 5. C Section B 6. B 7. A 8. C 9. A 10. B 11. C 12. A 13. A 14. B1 5. C Section C 16. B 17. A 18. C 19. C 20. B Section D 21. professor 22. presented 23. brain 24. criminal 25. intelligence tests 26. removed from 27. water pipes 28. blood test 29. in jail 30. aggression and learning Part II Vocabulary and Structure 31. D 32. A 33. D 34. B 35. A 36. D 37. C 38. B 39. C 40. B 41. B 42. A4 3. A 44. C 45. D Part III Reading Comprehension Section A 46. B47. C48. A49. D Section B 50. Y 51. N 52. NG 53. N 54. Y5 5. Y 56. individual rooms on the tour 57. umbrellas, wallets, cell phones and car keys 58. public transportation / subway 59. public telephones Section C 60. Water. 61. Nitrogen. / The presence of nitrogen. 62. The search for water. 63. The small amount (2% to 3%) of nitrogen in the Martian atmosphere . 第 63 页 共 77 页 2003-2010年全国大学生英语竞赛C类初赛、决赛真题及答案汇总集(原创) 64. Because current instruments aren’t very sensitive. Section D 65. separate movements 66. secure this connection 67. Ludwig Guttmann 68. international event 69. Special Olympics 70. organizations Part IV Cloze Section A 71. personality 72. singer 73. leading 74. famous 75. industrialist Section B 76. term 77. medical 78. reduce 79. better 80. identify 81. examine 82. consider 83. speech 84. perform 85. respond Part V Translation Section A 86. 中国正迅速进入老龄化社会,目前存在的失业和人口过剩问题在十年之内 将转变成新的问,即劳动力短缺和孩子少老年人多的问题。 87. 其部分原因是缺少将基础理论和实践技能相结合的学校和过分关注通过高 考精英选拔考试。 88. 只有10%—25%的毕业生有资格被跨国企业雇用,其部分原因就是语言问 题;这一点被我所见过的每位在中国或印度工作的人力资源经理都认同。 89. 中国和印度刚刚取得学士学位的学生就想马上读硕士,而且他们自己以及 他们的父母都不理解实践经验的价值。 Section B 90. If you tell them the truth, you will be able to obtain this opportunity without being misunderstood by them. 91. I will try to accomplish the translation of the literature in time no matter how hard it is!. 92. Nobody can deny the fact that China has made great achievements in raising living standards of rural residents. Part VI IQ Test 93. 3245 94. 10 95. 1/16 96. shorter than 97. 8 Part VII Writing Task I NOTICE We are very honored to have Prof. Richard Gilbert from Harvard University to give us a lecture on American Liberal Education. Prof. Gilbert, who graduated from Standford in 1979 and received his Ph.D. in Education from Harvard in 1984, has been conducting research and teaching international students in Harvard for more than 20 years. He has become a leading expert in this field and has published several books and numerous papers on liberal education and education policy. His humorous and thought provoking speech will surely benefit all the audience. 第 64 页 共 77 页 2003-2010年全国大学生英语竞赛C类初赛、决赛真题及答案汇总集(原创) The lecture will be given in the English Department Conference Hall from 2:30 to 4:30. p.m., Friday afternoon, November 16, 2007. All the teachers and students are welcome. English Department Task II The Anti-Addiction System Now in China, the adoption of the anti-addiction system and a “real name checking” system in netbars has led to a heated debate among students. Some say that students’ self-discipline is very limited and now at least 14 percent of the urban teenage players are addicted to the Internet and some children even repeatedly disappear from home to spend their nights at local Internet cafes, and play truant from school also. Therefore, they have failed quite a few school exams. In this sense, the new policy can prevent minors developing online addiction. Others claim that the anti-addiction system is only a means to an end and cannot remove the root of the online addiction completely and they believe the key is to guide the minors to improve their self-control and balance their work and play. In my opinion, the latter point hits home for me because it cuts to the chase. However, education is a long process and takes time, which means before education produces effects on children, some supplementary measures should also be taken to speed up the process and the anti-addiction system is one of them. 2008 National English Contest for College Students (Level C--- Preliminary) Part I Listening Comprehension (25 minutes, 30 marks) Section A (5 marks) In this section, you will hear five short conversations. At the end of each conversation, a question will be asked about what was said. Both the conversation and the question will be read only once. After each question, there will be a pause. During the pause, read the three choices marked A, B and C, and decide which is the best answer. Then mark the corresponding letter on the Answer Sheet with a single line through the center. 1. A. To buy a newspaper. B. To find a chemist. C. To post a letter. 2. A. Their billing system has been efficient. B. Their old billing system is outdated. C. He’s unimpressed with the new billing system. 3. A. She’s been on vacation. B. She’s been at the grocery store. C. She’s been on a business trip. 4. A. To go to the market. B. To have coffee. C. To have a discussion. 5. A. Changing a flight reservation. B. Canceling a hotel reservation. C. Reserving an extra hotel room. Section B (10 marks) In this section, you will hear two long conversations. Each conversation will be read only once. At the end of each conversation, there will be a one-minute pause. During the pause, read the five questions, each with three choices marked A, B and C, and decide which is the best answer. Then mark the corresponding letter on the Answer Sheet with a single line through the center. Conversation one 6. What’s Tania’s phone number? 第 65 页 共 77 页 2003-2010年全国大学生英语竞赛C类初赛、决赛真题及答案汇总集(原创) A. 506-6178 B. 501-6678 C. 501-6687 C. Karl’s mother. C. By tube. C. C. Rose Chianti. 7. Who is going shopping at Harrods? A. Tania. B. Karl. 8. How is Tania going to get to Harrods? A. On foot. B. By taxi. 9. Which street plan shows the way from the station? A. B. 10. What kind of wine does Karl want? A. Red Merlot. B. White Bordeaux. Conversation Two 11. Which drawing shows the position of the handle? A. B. C. 12. What other product features are included? A. A range of handle sizes. B. A variety of color combinations. C. A set of rings for different-sized tins. 13. What’s the purpose of the V-clip? A. To hold the paint brush so it drips in the can. B. To prevent paint from getting onto the painted surfaces. C. To prevent the brush drying out if the painter is interrupted. 14. Who will probably use the can-holder? A. Artists. B. Decorators. C. Soldiers. C. $4. 15. How much does the product cost? A. ,4. B. ,5. Section C (5 marks) In this section, you will hear five short news item. Each item will be read only once. After each item, there will be a pause. During the pause, read the question and the three choices marked A, B and C, and decide which is the best answer. Then mark the corresponding letter on the Answer Sheet with a single line through the center. 16. What has prompted the new administration to review the Australia’s citizenship test? A. It’s out of date. B. Higher immigration rates. C. Many applicants failing it. 17. Which train provides the easiest way to see Mount Fuji? A. From Tokyo to Yokohama. B. From Osaka to Yokohama. C. From Tokyo to Osaka. 18. How many U.N. employees were killed in 2007? A. 17. B. 42. C. 59. 19. What have studies found out about severe pneumonia treatment? A. No medication works in developing countries. B. It can be effectively treated at home with oral antibiotics. C. It can only be treated in the hospital with injectable antibiotics. 20. What was the bar’s business like before the smoking ban? A. Almost empty every day. B. Usually crowded at midday. C. Empty on Wednesdays. Section D (10 marks) In this section, you will hear a short passage. There are ten missing words or phrases. Fill in the 第 66 页 共 77 页 2003-2010年全国大学生英语竞赛C类初赛、决赛真题及答案汇总集(原创) blanks with the exact words you hear on the tape. Remember to write the answers on the Answer Sheet. Crime isn’t rising because the laws are not strict enough, or because the (21) ________ are not tough enough, or simply because there aren’t enough policemen on the beat. It is rising because the agencies and individuals who can have a (22) _______ effect on crime are not pulling together or in the same (23) ______. The police are one of those agencies, of course. But crime prevention is also a matter for (24) ______ who are content to see mass (25) _______ turn into a way of life, for councils who prefer to shout yah-boo at the police, (26) _____ getting on with making life safer for local people, for architects who design exposed and indefensible buildings, for parents who don’t take enough interest in what their teenage children are getting up to and for (27) ______ who sit inside and moan (28) ______ do anything about the state of the neighborhood or the street. Crime prevention has been neglected for too many by too many. Over the last ten years it has been willfully neglected in defiance of mounting (29) _______ that it is the central hope of (30) ________ action against crime. Part II Vocabulary and Structures (10 minutes, 15 marks) There are 15 incomplete sentences in this part. For each blank there are four choices, marked A, B C and D. Choose the one that best completes the sentence, then mark the corresponding letter on the Answer Sheet with a single line through the center. 31. They are looking for _______ man with _______ long dark hair. He is armed and dangerous. A. a; a B. the; a C. a; / D. /; the 32. It was unfortunate, but she had no _____ but to act as she did. A. chance. B. opportunity C. option D. solution 33. The _____ driver thinks that accidents only happen to other people. A. general B. usual C. common D. average 34. How did they manage to steal the Van Gogh? It was right ____ the security guard’s nose. A. below B. before C. under D. beside 35. The student ______ continuing his studies when he had to return to his home country unexpectedly. A. is considering B. was considering C. should consider D. has considered 36. My friends and I enjoy doing many of the same things. In that respect, we have a lot _______. A. in similar B. in particular C. in common D. in accordance 37. Our planned visit to the United States _______ because we were unable to get the visas. A. fell over B. fell down C. fell through 38. Lest anyone _____ it strange, let me assure you that it is quite true. D. fell on A. thought B. should think C. had thought D. thinking 39. When Sally ______ the criminal ______ her house, she screamed at the top of her voice because she didn’t want ______. A. saw; enter; killed C. sees; entering; be killed A. who B. saw; enter; to be killed D. sees; enter; killing C. which D. as 40. Tina’s children, _____ all live nearby, organized a big party for her eightieth birthday. B. that 41. Sometimes it is necessary to be careful _______ the right date to sit for a test. A. when choosing B. when you will choose C. when you have chosen D. when you chose 第 67 页 共 77 页 2003-2010年全国大学生英语竞赛C类初赛、决赛真题及答案汇总集(原创) 42. Einstein’s theory of relativity seemed hard to believe at the time _______. A. when he first introduced B. that he first introduced it C. he first introduced D. which he first introduced it 43. ________ really. I like both public transport and driving. A. On second thoughts B. I could go either way C. I will never learn to drive A. Can you keep a secret? C. Many happy returns. D. But then again B. Well I never! D. Congratulations! 44. _____ I’m getting married! 45. --- So, do you know where you’d like to go? --- Actually, I’ve heard that it’s very expensive and it’s cold all the time. A. I beg your pardon? B. Yeah, but it’s not very good. C. Do you know any good hotels? D. Do you have any suggestions? Part III Reading Comprehension (20 minutes, 40 marks) Section A (4 marks) There is one passage in this section with 4 questions. For each question, there are four choices marked A, B C and D. You should decide on the best choice, then mark the corresponding letter on the Answer Sheet with a single line through the center. Questions 46-49 are based on the following passage. Most sore throats are caused by an infection which treatment with antibiotics cannot cure. But with simple remedies the patient normally gets better in 4 or 5 days. Tonsillitis, however, usually starts with a sore throat which causes pain on swallowing. With children and some adults there may be a fever and the patient is obviously not feeling well. It may be possible to see white spots on the back of the throat. The neck may also swell, both of which are the normal response to infection. Sometimes a sore throat may occur with the common cold, and with influenza there may be dryness of the throat, pain on coughing and loss of voice. TREATMENT Aspirin: To help relieve the pain on swallowing and (if there is one) the fever, use aspirin tablets dissolved in water, so that the patient can gargle before swallowing. Repeat the treatment every 4 hours. Drink: Encourage the patient to drink plenty of liquids. Food: Food should not be forced on a patient who does not want to eat. Steam: If there is pain in the throat on coughing, breathing in steam may help. CHILDREN Young children, who may not be able to gargle, should be given aspirin dissolved in water every 4 hours at the right dose for their age. At one year: a single junior aspirin. At five years: half an adult aspirin. At eight years: one whole adult aspirin. WHEN TO SEE THE DOCTOR If the sore throat is still getting worse after 2 days. If the patient complains of earache. If the patient or parent is very worried. 第 68 页 共 77 页 2003-2010年全国大学生英语竞赛C类初赛、决赛真题及答案汇总集(原创) 46. According to the passage, it would appear that most sore throats _________. A. B. C. D. require an immediate visit to a doctor respond quickly to treatment with an antibiotic rarely turn out to be symptoms of serious illness result in tonsillitis even when treated 47. In order to treat a sore throat one should ________. A. prevent the patient from eating too much B. give the patient up to 4 aspirin tablets every day C. advise the patient to drink large amounts of liquids D. make the patient gargle with hot liquids 48. You should call the doctor in if _________. A. swelling occurs in the region of the ears B. the patient’s condition worsens after 2 days C. the patient’s throat is still sore after a week D. the disease spreads to another member of the family 49. What difference is there in the way adults and young children should be treated with aspirin? A. Young children should not be allowed to gargle with it. B. Adults should be given tablets to swallow whole. C. Young children should be given aspirin more often than adults. D. Adults should be given larger doses of aspirin than children. Section B (14 marks) There is one passage in this section with 10 questions. Go over the passage quickly and answer the questions on the Answer Sheet. For questions 50-55, mark Y (for YES) if the statement agrees with the information given in the passage; N (for NO) if the statement contradicts the information given in the passage; NG (for NOT GIVEN) if the information is not given in the passage. For questions 56-59, complete the sentences with the information given in the passage. In 1945 leaders from 51 countries met in San Francisco, California, and organized the United Nations (often called the UN). World War II had just ended. Millions of people had died, and there was destruction everywhere. People hoped they could build a future of world peace through this new organization. The United Nations has four main goals and purposes: 1. To work together for international peace and to solve international problems; 2. To develop friendly relations among nations; 3. To work together for human rights for everyone of all races, religions, and of both sexes; 4. To build a center where nations can work together for these goals. Today almost every country in the world is a member of the UN. Each country has signed an agreement that says: 1. All members are equal. 2. All members promise to solve international problems in a peaceful way. 3. No member will use force against another member. 4. All members will help the UN in its actions. 5.The UN will not try to solve problems within countries except to enforce international peace. 第 69 页 共 77 页 2003-2010年全国大学生英语竞赛C类初赛、决赛真题及答案汇总集(原创) Obviously, the United Nations has not been completely successful in its goals. There have been several wars since 1945. However, the organization has helped bring peace to some countries that were at war. It has helped people who left their countries because of wars. It has helped bring independence to colonies. The main United Nations organization is in New York City, but the UN has a “family” of other related organizations all over the world. These organizations try to provide a better life for everyone. One example is UNICEF, an organization that provides food, medical care, and many other services for poor children wherever they live. Another example is the World Health Organization, which develops medical programs all over the world. There are thousands of UN workers in developing countries. They work as planners to increase production in farming and industry. They provide medical services, improve education programs, and spread scientific information. They develop programs that provide jobs and better living conditions. They help countries control their population growth. The United Nations also holds large international conferences, where people meet to discuss important world issues. One conference was about the uses and ownership of oceans. Another was about women. There are also International Years. In these special years, people work together to improve situations and solve problems. One year was the International Year of the Child. For centuries countries have fought each other, and powerful countries have taken control of weaker ones. It is very difficult to persuade nations to live together in peace. Nevertheless, the United Nations is attempting to do this. It is the only organization that unites the world in the search for peace. Section C (10 marks) In this section, there is one passage followed by 5 questions. Read the passage carefully, and then answer the questions in as few words as possible (not more than 10 words). Remember to write the answers on the Answer Sheet. Questions 60-64 are based on the following passage. A language is a system of sounds, gestures, or characters used by humans to communicate their ideas and feelings. There are about 4,000 spoken languages in the world. Some are spoken by millions of people. Other languages have only a few speakers. All languages have rules for forming words and for ordering those words in sentences. In written languages, meaning is expressed through a system of characters and rules for combining those characters. In spoken languages, meaning is expressed through a system of sounds and rules for combining those sounds. Many deaf people use sign languages, in which gestures do the work of the sound system of spoken languages. Word order is more important in English than it is in some other languages, such as Russian. The sound system is very important in Chinese and in many languages spoken in West Africa. Languages are always changing, but they change very slowly. People invent new words for their language, borrow words from other languages, and change the meanings of words as needed. For example, the English word byte was invented by computer specialists in 1959. the English word tomato was borrowed from Nahuatl, and American Indian language spoken in Mexico. The English word meat once referred to food in general. There are several major language families in the world. The languages in each family are related, and scientists think they came from the same parent family. Language families come in 第 70 页 共 77 页 2003-2010年全国大学生英语竞赛C类初赛、决赛真题及答案汇总集(原创) different sizes. The Austronesian family contains at least 500 languages, including Pilipino, Malay, and Maori. The Basque language, spoken in northern Spain, is the only member of its language family. The Indo-European language family contains 55 languages, including English, German, Spanish, Russian, and Hindi. Another language family is Sino-Tibetan, which includes Chinese, Burmese, and Tibetan. The Afro-Asiatic family includes Arabic, Hebrew, and Amharic. There are about 150 American Indian languages spoken today. These languages have many differences and have been divided into more than 50 language families. People learn languages by listening, copying what they hear, and using the language. Most children learn their first languages easily, and sometimes other languages as well. Adults often must work harder at learning a second language. Questions: Section D (12 marks) In this section, there is one passage followed by a summary. Read the passage carefully and complete the summary below by choosing a maximum of three words from the passage to fill in each of the spaces, 65-70. Remember to write the answers on the Answer Sheet. Questions 65-70 are based on the following passage. Welcome to “Software World” --- bringing you the very latest information on what is currently available on CD-ROM. Are you a director or producer looking for an unusual actor to play a part in a new movie project, or with that special look for a new commercial on TV? OK. The usual procedure would be to contact a theatrical agency, who would try and sell you the idea of using one of the actors listed on their books. Books? Too old-fashioned for you? Then get yourself a copy of this latest electronic database called “The Electronic Curtain”. The brainchild of casting agent Fred Harkney of the Better Talent Agency, he says he got the idea of an actor’s directory from his son playing computer games. Noting that junior had to type in the details of the characters in his favorite game, he realized he could do the same for the actors he represents. Eventually, he came to include information on nearly three quarters of the approximately 34,000 actors registered and looking for work in Britain. With some theatrical agencies boasting that they represent over 500 performers, the need for this product is not hard to fathom. It can be a nightmare trying to remember just which actor has done what, or just what an actor can do. The database lists details of over 5,500 actors: TV shows they have appeared in, special skills they possess --- everything down to the color of their eyes and other distinguishing physical features. By entering the details of the type of person you are looking for, the database quickly locates only those persons with the particular qualities requested. One problem though, is that many actors feel it is too impersonal, and they could be missing out on much needed auditions for parts in theatrical shows. On the other hand, it might just get them that elusive job. The days of nervous nail-biting while waiting around to give an audition could well and truly be over. And all because of a small plastic disk. For product details ring this number now: 0171-379-60000. Summary: Part IV Cloze (15 minutes, 15 marks) 第 71 页 共 77 页 2003-2010年全国大学生英语竞赛C类初赛、决赛真题及答案汇总集(原创) Section A (5 marks) There are 5 blanks in the passage. Use the word given on the right-hand side to form a word that fits in each blank. Remember to write the answers on the Answer Sheet. Section B (10 marks) There are 10 blanks in the passage. For each blank, one or more letters of the word have been given (not exceeding 3 letters). Read the passage below and decide which word best fits each blank, using only one word in each word in each blank. Remember to write the answers on the Answer Sheet. True relaxation is most certainly not a matter of flopping down in front of the television with a welcome drink. Nor is it about drifting into an exhausted sleep. Useful though these responses to tension and over-tiredness (76) mi_______ be, we should distinguish between them and conscious relaxation in (77) te_____ of quality and effect. Regardless of the level of tiredness, real relaxation is a state of alertness, yet, at the same (78) t______, passive awareness, in which our bodies are (79) a________ rest while our minds are awake. Moreover, it is as natural for a healthy person to be relaxed when moving as resting. (80) Be_____ relaxed in action means we bring the appropriate energy to everything we do, (81) W______ needs to be rediscovered is conscious relaxation. With this in (85) m______ we must apply ourselves to understanding stress and the nature of its causes, however deep-seated. Part V Translation (15 minutes, 15 marks) Section A (8 marks) Translate the underlined sentences of the following passage into Chinese. Remember to write the answers on the Answer Sheet. After my mother passed away, my dad tried even harder to stay healthy and active. Every morning, until the weather turned too cold, he swam in the turquoise pool in the complex where he lived. (86) Each day, no matter how he felt, he swam one more lap than the previous day, just to prove there was always room for improvement. Every few days he reported the new number of laps to me, pride edging his voice. By his late seventies, in spite of swimming and working six days a week, my dad had noticeably dwindled in strength and energy. By age eighty-one he was in poor health and had to retire. (87) He pretended he didn’t need to lean heavily on me for support as we walked slowly, and I pretended not to notice. His mind was clear, but congestive heart problems and disabling arthritis had worn him down. One day he said, “In case of an emergency, I do not wish to be kept alive by any extraordinary means. I’ve signed an official paper to this effect.” He smiled his wonderful, broad grin and said, “I’ve been blessed to have had your mother as my wife and you as my only child, and I’m ready to go.” Less than a month later he had a heart attack. In the emergency room, he again reminded his doctor and me of his wishes, but I couldn’t imagine, in spite of this latest crisis, that he wouldn’t always be saying, “Have I told you yet today that I adore you?” (88) He was miserable in intensive care; tubes seemed to come from every opening. But my dad still had his sense of humor, asking me, “Does this mean we can’t keep our lunch date tomorrow?” His voice faltered. “I’ll be here to pick you up and we’ll go someplace special,” I answered, a lump in my throat. (89) Dad refused to look at me for the first time in his life and turned toward the blank green 第 72 页 共 77 页 2003-2010年全国大学生英语竞赛C类初赛、决赛真题及答案汇总集(原创) wall next to his hospital bed. There was a painful silence between us. He said, “I don’t want you to remember me like this. Promise me you won’t, darling! And please go now --- I’m so miserable. Section B (7 marks) Translate the following sentences into English, using the words given in the brackets. Remember to write the answers on the Answer Sheet. 90. 她在阳光下只晒了一会儿,脸就开始变红了。(exposure) 91. 我情愿你没有对任何人提及此事。(would rather) 92. 采纳新规则的建议是主席提出的。(suggestion, 同位语从句) Part VI IQ Test (5 minutes, 5 marks) There are 5 IQ questions in this part. Write the answers on the Answer Sheet. 93. Find the missing number. 7 9 8 10 9 ? 94. Find a nine-letter word by moving from letter to letter in any direction. Each letter must only be used once. A K N B I R D U O C L D Z M T E Clue: OK to drink. 95. Complete the sequence. A C A E A G A K A M A ? 96. How may different arrangements can you make of all the seven letters in the word COMBINE? 97. Change just one letter in each word to find a familiar phrase. MALE FOOD Part VII Writing (30 minutes, 30 marks) Task I (10 marks) You are planning to take a computer course with a friend who has sent you a letter with a school advertisement. Read the letter and the advertisement, together with the notes. Then write to the school, asking for the information which your friend suggests and adding any relevant questions of your own. Write a letter of between 80 and 100 words in an appropriate style on the Answer Sheet. Do not write any address. Task II (20 marks) Write about the following topic: Do you agree or disagree? Give reasons for your answer and include any relevant examples from your own knowledge or experience. Write between 100 and 120 words on the Answer Sheet. 参考答案 Part I Listening Comprehension (25 minutes, 30 marks) Section A (5 marks) 1. B 2. B 3. A 4. C 5. C Section B (10 marks) 6. B 7. A 8. C 9. C 10. A 11. A 12. C 13. A 14. B 15. A 第 73 页 共 77 页 2003-2010年全国大学生英语竞赛C类初赛、决赛真题及答案汇总集(原创) Section C (5 marks) 16. C 17. C 18. B 19. B 20. B Section D (10 marks) 21. sentences 22. preventative 23. direction 24. governments 25. unemployment 26. instead of 27. householders 28. rather than 29. evidence 30. effective Part II Vocabulary and Structures (10 minutes, 15 marks) 31. C 32. C 33. D 34. C 35. B 36. C 37. C 38. B 39. B 40. A 41. A 42. B 43. B 44. A 45. D Part III Reading Comprehension (20 minutes, 40 marks) Section A (4 marks) 46. C 47. C 48. B 49. D Section B (14 marks) (注:50-55题,每题1分;56-59 题,每题2分。) 50. Y 51. Y 52. NG 53. N 54. NG 55. N 56. will use force 57. completely successful 58. large international conferences 59. attempting to persuade Section C (10 marks) 60. About 4,000. 61. For forming words and for ordering those words in sentences. 62. By gestures. 63. Word order. 64. The Basque language. Section D (12 marks) 65. three quarters of 66. theatrical agencies 67. TV shows 68. particular qualities 69. too impersonal 70. product details Part IV Cloze (15 minutes, 15 marks) Section A (5 marks) 71. imperfect 72. selection 73. Similarly 74. confidence 75. honesty Section B (10 marks) 76. might 77. terms 78. time 79. at 80. Being 81. so 82. rather 83. consequence 84. What 85. mind Part V Translation (15 minutes, 15 marks) Section A (8 marks) 86. 不论感觉怎样,他每天都比前一天多游一圈,以此证明每次都有提高的余地。 87. 一起慢走时父亲装出一副不十分需要我搀扶的样子,而我也佯装不见。 88. 重病房94. DRINKABLE 95. Q 96. 5040 97. MAKE GOOD Part VII Writing (30 minutes, 30 marks) Task I (10 marks) 参考范文略 第 74 页 共 77 页 2003-2010年全国大学生英语竞赛C类初赛、决赛真题及答案汇总集(原创) Task II (20 marks) 参考范文略 2009 National English Contest for College Students (Level C - Preliminary) 参考答案及作文评分 标准 excel标准偏差excel标准偏差函数exl标准差函数国标检验抽样标准表免费下载红头文件格式标准下载 Part I Listening Comprehension (25 minutes, 30 marks) Section A (5 marks) 1. B 2. B 3. C 4. A 5. C Section B (10 marks) 6. A 7. B 8. C 9. C 10. B 11. A 12. C 13. B 14. A 15. C Section C (5 marks) 16. C 17. B 18. C 19. B 20. C Section D (10 marks) 21. losing 22. minimise 23. expenditures 24. vocational 25. available 26. previously 27. complex 28. insurance 29. security and freedom 30. heavily involved Part II Vocabulary and Structure (10 minutes, 15 marks) 31. A 32. D 33. B 34. C 35. D 36. C 37. B 38. C 39. D 40. C 41. C 42. B 43. D 44. B 45. B Part III Reading Comprehension (20 minutes, 35 marks) Section A (5 marks) 46. B 47. A 48. A 49. D 50. D Section B (10 marks) 51. the family name 52. surprising 53. economic reasons 54. dowries 55. individuals Section C (10 marks) 56. 2009. 57. It’s a professional body representing managers and personnel staff. 58. Survival. 59. Find new ways to motivate their employees to perform. 60. To help preserve staff loyalty and engagement. Section D (10 marks) 61. average 62. has been connected / is connected 63. skin 64. nutrients 65. crucial Part IV Cloze (15 minutes, 15 marks) 66. speaking 67. encouraged 68. cheapest 69. composer 70. between 71. especially 72. like / including 73. hundreds 74. when 75. and 76. belief 77. exploration 78. developing / designing 79. program(me) 80. over Part V Translation (15 minutes, 20 marks) Section A (10 marks) 81. 从皇帝身上华丽的龙袍到当今的时装,刺绣给我们的文化和生活增添了许 多乐趣。 82. 直到后来,随着国家经济的发展,刺绣才进入平常百姓的生活。 83. 马王堆汉墓出土的文物是这个时期刺绣空前发展的最好见证。 84. 汉语“刺绣”一词中的“绣”,意思是由五种颜色组成的图画或刺绣,它暗示 着美丽壮观的事物。 85. 倾注了智慧和热情,刺绣确实是大家闺秀用来打发时间的一种很好的消遣。 Section B (10 marks) 86. She was disabled from birth, but never did she give in to any difficulty. 第 75 页 共 77 页 2003-2010年全国大学生英语竞赛C类初赛、决赛真题及答案汇总集(原创) 87. Whenever you say anything, you must at least make yourself understood. 88. It is important for a business to keep pace with changes in the market. 89. If you book your trip through a travel agency, you’re likely to get a big discount. 90. Had we invested in the telecommunications industry, we would be quite rich by now. Part VI IQ Test (5 minutes, 5 marks) 91. G. 92. Four spades. 93. QZQDS. 94. E resort. 95. ZN. Part VII Writing (30 minutes, 30 marks) 参考范文略。 Task I 应用文评分标准 一、评分原则: 1. 本题满分为10分。 2. 评分时,先根据文章的内容和语言初步确定其所属档次,然后以该档次的要求来衡量、确定或调整本档次,最后给分。 3. 词数少于80的,从总分中减去2分。 4. 如 关于书的成语关于读书的排比句社区图书漂流公约怎么写关于读书的小报汉书pdf 写较差,以致影响阅卷,将分数降低一个档次。 二、各档次的给分范围和要求: 第四档:8—10分 完全符合写作格式的要求,覆盖所有内容要点,表达思想清楚,文字通顺,连贯性很好,基本上无词汇和语法错误。 第三档:6—7分 基本符合写作格式的要求,有个别地方表达思想不够清楚,文字基本通顺、连贯,有少量词汇和语法错误。 第二档:3—5分 未恰当完成写作格式的要求,漏掉内容要点,表达思想不清楚,文字多处出现词汇和语法错误,影响了对写作内容的理解。 第一档:1—2分 未完成写作格式的要求,明显遗漏主要内容,表达思想紊乱,有较多词汇和语法的重大错误,未能将信息传达给读者。 0分 白卷;作文与题目毫不相关;内容太少,无法评判;所写内容无法看清。 Task II 议论文评分标准 一、评分原则: 1. 本题满分为20分。 2. 评分时,先根据文章的内容和语言初步确定其所属档次,然后以该档次的要求来衡量、确定或调整本档次,最后给分。 3. 词数少于120的,从总分中减去2分。 4. 如书写较差,以致影响交流,将分数降低一个档次。 二、各档次的给分范围和要求: 第四档:16—20分 完全符合写作格式的要求,覆盖所有内容要点,表达思想清楚,文字通顺,连贯性很好,基 第 76 页 共 77 页 2003-2010年全国大学生英语竞赛C类初赛、决赛真题及答案汇总集(原创) 本上无词汇和语法错误。 第三档:11—15分 基本符合写作格式的要求,有个别地方表达思想不够清楚,文字基本通顺、连贯,有少量词汇和语法错误。 第二档:6—10分 未恰当完成写作格式的要求,漏掉内容要点,表达思想不清楚,文字多处出现词汇和语法错误,影响了对写作内容的理解。 第一档:1—5分 未完成写作格式的要求,明显遗漏主要内容,表达思想紊乱,有较多词汇和语法的重大错误,未能将信息传达给读者。 0分 白卷;作文与题目毫不相关;内容太少,无法评判;所写内容无法看清。 第 77 页 共 77 页
本文档为【2003-2011年全国大学生英语竞赛C类初赛、决赛真题及答案汇总】,请使用软件OFFICE或WPS软件打开。作品中的文字与图均可以修改和编辑, 图片更改请在作品中右键图片并更换,文字修改请直接点击文字进行修改,也可以新增和删除文档中的内容。
该文档来自用户分享,如有侵权行为请发邮件ishare@vip.sina.com联系网站客服,我们会及时删除。
[版权声明] 本站所有资料为用户分享产生,若发现您的权利被侵害,请联系客服邮件isharekefu@iask.cn,我们尽快处理。
本作品所展示的图片、画像、字体、音乐的版权可能需版权方额外授权,请谨慎使用。
网站提供的党政主题相关内容(国旗、国徽、党徽..)目的在于配合国家政策宣传,仅限个人学习分享使用,禁止用于任何广告和商用目的。
下载需要: 免费 已有0 人下载
最新资料
资料动态
专题动态
is_037433
暂无简介~
格式:doc
大小:478KB
软件:Word
页数:0
分类:工学
上传时间:2018-01-05
浏览量:18